Chapter 22: Transition to Parenthood*, Chapter 21 Nursing Care of the Family During Postpartum Period Lowdermilk*, Chapter 33: Postpartum Complications, Lowdermilk 11th Ed. Chapter 32: Labor and Birth Complications, Lowdermilk-Ch. 20

Pataasin ang iyong marka sa homework at exams ngayon gamit ang Quizwiz!

Which statement by a newly delivered woman indicates that she knows what to expect regarding her menstrual activity after childbirth? a. "My first menstrual cycle will be lighter than normal and then will get heavier every month thereafter." b. "My first menstrual cycle will be heavier than normal and will return to my prepregnant volume within three or four cycles." c. "I will not have a menstrual cycle for 6 months after childbirth." d. "My first menstrual cycle will be heavier than normal and then will be light for several months after."

"My first menstrual cycle will be heavier than normal and will return to my prepregnant volume within three or four cycles." "My first menstrual cycle will be heavier than normal and will return to my prepregnant volume within three or four cycles" is an accurate statement and indicates her understanding of her expected menstrual activity. She can expect her first menstrual cycle to be heavier than normal, which occurs by 3 months after childbirth, and the volume of her subsequent cycles will return to prepregnant levels within three to four cycles.

A woman is admitted to the birth setting in early labor. She is 3 cm dilated, -2 station, with intact membranes and FHR of 150 beats/min. Her membranes rupture spontaneously, and the FHR drops to 90 beats/min with variable decelerations. What would the initial response from the nurse be? 1. Perform a vaginal exam. 2. Notify the physician. 3. Place the client in a left lateral position. 4. Administer oxygen at 2 L per nasal cannula.

1 Explanation: 1. A drop in fetal heart rate accompanied by variable decelerations is consistent with a prolapsed cord. The nurse would assess for prolapsed cord via vaginal examination.

The nurse is making client assignments for the next shift. Which client is most likely to experience a complicated labor pattern? 1. 34-year-old woman at 39 weeks gestation with a large-for-gestational-age (LGA) fetus 2. 22-year-old woman at 23 weeks gestation with ruptured membranes 3. 30-year-old woman at 41 weeks gestation and estimated fetal weight 7 pounds 8 ounces 4. 43-year-old woman at 37 weeks gestation with hypertension

1 Explanation: 1. A risk factor for hypotonic uterine contraction patterns includes a large-for-gestational-age (LGA) fetus.

On assessment, a labor client is noted to have cardiovascular and respiratory collapse and is unresponsive. What should the nurse suspect? 1. An amniotic fluid embolus 2. Placental abruption 3. Placenta accreta 4. Retained placenta

1 Explanation: 1. Cardiovascular and respiratory collapse are symptoms of an amniotic fluid embolus and cor pulmonale.

What would be a normal cervical dilatation rate in a first-time mother (primip)? 1. 1.5 cm per hour 2. Less than 1 cm cervical dilatation per hour 3. 1 cm per hour 4. Less than 0.5 cm per hour

1 Explanation: 1. Dilatation in a multip is about 1.5 cm per hour.

The client gave birth to a 7 pound, 14 ounce female 30 minutes ago. The placenta has not yet delivered. Manual removal of the placenta is planned. What should the nurse prepare to do? 1. Start an IV of lactated Ringers. 2. Apply anti-embolism stockings. 3. Bottle-feed the infant. 4. Send the placenta to pathology.

1 Explanation: 1. In women who do not have an epidural in place, intravenous sedation may be required because of the discomfort caused by the procedure. An IV is necessary.

A woman has been having contractions since 4 a.m. At 8 a.m., her cervix is dilated to 5 cm. Contractions are frequent, and mild to moderate in intensity. Cephalopelvic disproportion (CPD) has been ruled out. After giving the mother some sedation so she can rest, what would the nurse anticipate preparing for? 1. Oxytocin induction of labor 2. Amnioinfusion 3. Increased intravenous infusion 4. Cesarean section

1 Explanation: 1. Oxytocin is the drug of choice for labor augmentation or labor induction and may be administered as needed for hypotonic labor patterns.

Two hours ago, a client at 39 weeks gestation was 3 cm dilated, 40% effaced, and +1 station. Frequency of contractions was every 5 minutes with duration 40 seconds and intensity 50 mmHg. The current assessment is 4 cm dilated, 40% effaced, and +1 station. Frequency of contractions is now every 3 minutes with 40-50 seconds duration and intensity of 40 mmHg. What would the priority intervention be? 1. Begin oxytocin after assessing for CPD. 2. Give terbutaline to stop the preterm labor. 3. Start oxygen at 8 L/min. 4. Have the anesthesiologist give the client an epidural.

1 Explanation: 1. The client is having hypertonic contractions. Cephalopelvic disproportion (CPD) must be excluded. If CPD exists, oxytocin (Pitocin) augmentation should not be used. Oxytocin is the drug of choice for labor augmentation or labor induction.

What is one of the most common initial signs of nonreassuring fetal status? 1. Meconium-stained amniotic fluid 2. Cyanosis 3. Dehydration 4. Arrest of descent

1 Explanation: 1. The most common initial signs of nonreassuring fetal status are meconium-stained amniotic fluid and changes in the fetal heart rate (FHR).

A 26-year-old client is having her initial prenatal appointment. The client reports to the nurse that she suffered a pelvic fracture in a car accident 3 years ago. The client asks whether her pelvic fracture might affect her ability to have a vaginal delivery. What response by the nurse is best? 1. It depends on how your pelvis healed. 2. You will need to have a cesarean birth. 3. Please talk to your doctor about that. 4. You will be able to delivery vaginally.

1 Explanation: 1. Women with a history of pelvic fractures may also be at risk for cephalopelvic disproportion (CPD).

The nurse is caring for a client who could be at risk for uterine rupture. The nurse is monitoring the fetus closely for which of the following? Note: Credit will be given only if all correct choices and no incorrect choices are selected. Select all that apply. 1. Late decelerations 2. Bradycardia 3. Loss of ability to determine fetal station 4. Tachycardia 5. Early decelerations

1, 2, 3 Explanation: 1. Late decelerations could be seen with uterine rupture. 2. Bradycardia is seen if there is uterine rupture. 3. The uterus is not holding the fetus in place anymore if the uterus ruptures.

The nurse caring for a client in labor anticipates fetal macrosomia and shoulder dystocia. Appropriate management of shoulder dystocia is essential in order to prevent which fetal complications? Note: Credit will be given only if all correct choices and no incorrect choices are selected. Select all that apply. 1. Brachial plexus injury 2. Fractured clavicle 3. Asphyxia 4. Neurological damage 5. Puerperal infection

1, 2, 3, 4 Explanation: 1. Brachial plexus injury occurs due to improper or excessive traction applied to the fetal head. 2. Complications in macrosomia include fractured clavicles. 3. Complications in macrosomia include asphyxia of the fetus. 4. Neurological damage is a complication of macrosomia.

In caring for a client with a uterine rupture, the nurse determines which nursing diagnoses to be appropriate? Note: Credit will be given only if all correct choices and no incorrect choices are selected. Select all that apply. 1. Gas Exchange, Impaired 2. Fear related to unknown outcome 3. Coping, Ineffective 4. Mobility: Physical, Impaired 5. Anxiety

1, 2, 3, 5 Explanation: 1. Gas Exchange, Impaired diagnosis could apply to both mother and fetus. 2. The client would experience fear related to an unknown outcome. 3. Ineffective coping would be due to emergent situation secondary to uterine rupture. 5. There will be anxiety related to emergency procedures and unknown fetal outcome.

Risk factors for tachysystole include which of the following? Note: Credit will be given only if all correct choices and no incorrect choices are selected. Select all that apply. 1. Cocaine use 2. Placental abruption 3. Low-dose oxytocin titration regimens 4. Uterine rupture 5. Smoking

1, 2, 4 Explanation: 1. Cocaine use is a risk factor for tachysystole. 2. Placental abruption is a risk factor for tachysystole. 4. Uterine rupture is a risk factor for tachysystole.

During labor, the client at 4 cm suddenly becomes short of breath, cyanotic, and hypoxic. The nurse must prepare or arrange immediately for which of the following? Note: Credit will be given only if all correct choices and no incorrect choices are selected. Select all that apply. 1. Intravenous access 2. Cesarean delivery 3. Immediate vaginal delivery 4. McRoberts maneuver 5. A crash cart

1, 2, 5 Explanation: 1. When an amniotic fluid embolism is suspected, intravenous access is obtained as quickly as possible. 2. Shortness of breath, cyanosis, and hypoxia are symptoms of an amniotic fluid embolus, which necessitates immediate cesarean delivery. 5. The chances of a code are high, so the crash cart needs to be available.

What are the primary complications of placenta accrete? Note: Credit will be given only if all correct choices and no incorrect choices are selected. Select all that apply. 1. Maternal hemorrhage 2. Insomnia 3. Failure of the placenta to separate following birth of the infant 4. Autonomic dysreflexia 5. Shoulder dystocia

1, 3 Explanation: 1. The primary complications of placenta accreta are maternal hemorrhage and failure of the placenta to separate following birth of the infant. 3. The primary complications of placenta accreta are maternal hemorrhage and failure of the placenta to separate following birth of the infant.

Which instruction does the nurse give a postpartum client to prevent infections? 1. "Wipe from front to back after using the toilet." 2. "Use cold water to cleanse the perineal area." 3. "Change the perineal pad from back to front." 4. "Avoid the use of slippers while in the hospital."

1. "Wipe from front to back after using the toilet."

A client's chart indicates that she had a vaginal delivery and asks the nurse about the timing of her discharge home. What does the nurse inform the client about appropriate discharge timing? 1. "You are allowed to remain in the hospital for at least 48 hours after delivery." 2. "You will be discharged within 48 hours if there are complications." 3. "You can opt for discharge within 36 hours in case of complications." 4. "You will be discharged within 24 hours if there are no complications."

1. "You are allowed to remain in the hospital for at least 48 hours after delivery."

While assessing a postpartum client, the nurse finds that the client has a fourth-degree laceration. What immediate interventions should the nurse perform while caring for the client? 1. Apply an ice pack to limit edema during the first 12 to 24 hours 2. Instruct the patient to use two or more perineal pads 3. Teach the client to avoid taking sitz baths 4. Remind the client to avoid doing perineal (Kegel) exercises

1. Apply an ice pack to limit edema during the first 12 to 24 hours

While assessing a postpartum client early in the morning, the nurse finds that the client's perineal pad is completely saturated. What is the first step the nurse should take in this situation? 1. Ask the client when she last changed her perineal pad. 2. Inform the primary health care provider immediately. 3. Massage the client's uterine fundus vigorously. 4. Ask the night duty nurse to review the assessment.

1. Ask the client when she last changed her perineal pad.

Which food does the nurse suggest to the postpartum client to increase docosahexaenoic acid (DHA) in breast milk? 1. Fish 2. Eggs 3. Sugar 4. Citrus fruits

1. Fish

Which type of medication is used to decrease excessive bleeding and uterine atony in a postpartum client? 1. Oxytocic 2. Anesthetic 3. Antiinflammatory 4. Selective serotonin reuptake inhibitors

1. Oxytocic

Baby-friendly hospitals mandate that infants be put to breast how soon after birth? 1. Within the first 1 to 2 hours 2. Within the first 30 minutes 3. Within the first 2½ hours 4. Within the first 4 hours

1. Within the first 1 to 2 hours

During labor, the fetus was in a brow presentation, but after a prolonged labor, the fetus converted to face presentation and was delivered vaginally with forceps assist. What should the nurse explain to the parents? 1. The infant will need to be observed for meconium aspiration. 2. Facial edema and head molding will subside in a few days. 3. The infant will be given prophylactic antibiotics. 4. Breastfeeding will need to be delayed for a day or two.

2 Explanation: 2. Any facial edema and head molding that result from the use of forceps at birth will subside in a few days.

What is required for any women receiving oxytocin (Pitocin)? 1. CPR 2. Continuous electronic fetal monitoring 3. Administering oxygen by mask 4. Nonstress test

2 Explanation: 2. Continuous electronic fetal monitoring (EFM) is required for any women receiving oxytocin (Pitocin).

The client has undergone an ultrasound, which estimated fetal weight at 4500 g (9 pounds 14 ounces). Which statement indicates that additional teaching is needed? 1. Because my baby is big, I am at risk for excessive bleeding after delivery. 2. Because my baby is big, his blood sugars could be high after he is born. 3. Because my baby is big, my perineum could experience trauma during the birth. 4. Because my baby is big, his shoulders could get stuck and a collarbone broken.

2 Explanation: 2. Hypoglycemia, not hyperglycemia, is a potential complication experienced by a macrosomic fetus.

The nurse should anticipate the labor pattern for a fetal occiput posterior position to be which of the following? 1. Shorter than average during the latent phase 2. Prolonged as regards the overall length of labor 3. Rapid during transition 4. Precipitous

2 Explanation: 2. Occiput posterior (OP) position of the fetus is the most common fetal malposition and occurs when the head remains in the direct OP position throughout labor. This can prolong the overall length of labor.

The client delivered 30 minutes ago. Her blood pressure and pulse are stable. Vaginal bleeding is scant. The nurse should prepare for which procedure? 1. Abdominal hysterectomy 2. Manual removal of the placenta 3. Repair of perineal lacerations 4. Foley catheterization

2 Explanation: 2. Retention of the placenta beyond 30 minutes after birth is termed retained placenta. Manual removal of the placenta is then performed.

In succenturiate placenta, one or more accessory lobes of fetal villi have developed on the placenta, with vascular connections of fetal origin. What is the gravest maternal danger? 1. Cord prolapse 2. Postpartum hemorrhage 3. Paroxysmal hypertension 4. Brachial plexus injury

2 Explanation: 2. The gravest maternal danger is postpartum hemorrhage if this minor lobe is severed from the placenta and remains in the uterus.

A fetal weight is estimated at 4490 grams in a client at 38 weeks gestation. Counseling should occur before labor regarding which of the following? 1. Mothers undiagnosed diabetes 2. Likelihood of a cesarean delivery 3. Effectiveness of epidural anesthesia with a large fetus 4. Need for early delivery

2 Explanation: 2. The likelihood of a cesarean delivery with a fetus over 4000 grams is high. This should be discussed with the client before labor.

Dystocia encompasses many problems in labor. What is the most common? 1. Meconium-stained amniotic fluid 2. Dysfunctional uterine contractions 3. Cessation of contractions 4. Changes in the fetal heart rate

2 Explanation: 2. The most common problem is dysfunctional (or uncoordinated) uterine contractions that result in a prolongation of labor.

Nonreassuring fetal status often occurs with a tachysystole contraction pattern. Intrauterine resuscitation measures may become warranted and can include which of the following measures? Note: Credit will be given only if all correct choices and no incorrect choices are selected. Select all that apply. 1. Position the woman on her right side. 2. Apply oxygen via face mask. 3. Call for anesthesia provider for support. 4. Increase intravenous fluids by at least 700 mL bolus. 5. Call the physician/CNM to the bedside.

2, 3, 4 Explanation: 2. The nurse would apply oxygen via face mask. 3. The nurse would call for anesthesia provider for support. 4. The nurse would increase intravenous fluids by at least 500 mL bolus.

The nurse knows that the maternal risks associated with postterm pregnancy include which of the following? Note: Credit will be given only if all correct choices and no incorrect choices are selected. Select all that apply. 1. Polyhydramnios 2. Maternal hemorrhage 3. Maternal anxiety 4. Forceps-assisted delivery 5. Perineal damage

2, 3, 4, 5 Explanation: 2. Maternal symptoms and complications in postterm pregnancy may include maternal hemorrhage. 3. Maternal symptoms and complications in postterm pregnancy may include maternal anxiety. 4. Maternal symptoms and complications in postterm pregnancy may include an operative vaginal birth with forceps or vacuum extractor. 5. Maternal symptoms and complications in postterm pregnancy may include perineal trauma and damage.

Which of the following potential problems would the nurse consider when planning care for a client with a persistent occiput posterior position of the fetus? Note: Credit will be given only if all correct choices and no incorrect choices are selected. Select all that apply. 1. Increased fetal mortality 2. Severe perineal lacerations 3. Ceasing of labor progress 4. Fetus born in posterior position 5. Intense back pain during labor

2, 3, 4, 5 Explanation: 2. The woman can have third- or fourth-degree perineal laceration or extension of a midline episiotomy. 3. Sometimes labor progress ceases if the fetus fails to rotate to an occiput anterior position. 4. Occiput posterior positions are associated with a higher incidence of vacuum-assisted births. 5. The woman usually experiences intense back pain in the small of her back throughout labor.

Maternal risks of occiput posterior (OP) malposition include which of the following? Note: Credit will be given only if all correct choices and no incorrect choices are selected. Select all that apply. 1. Blood loss greater than 1000 mL 2. Postpartum infection 3. Anal sphincter injury 4. Higher rates of vaginal birth 5. Instrument delivery

2, 3, 5 Explanation: 2. Postpartum infection is a maternal risk of OP. 3. Anal sphincter injury is a maternal risk of OP. 5. Instrument delivery is a maternal risk of OP.

Risk factors for labor dystocia include which of the following? Note: Credit will be given only if all correct choices and no incorrect choices are selected. Select all that apply. 1. Tall maternal height 2. Labor induction 3. Small-for-gestational-age (SGA) fetus 4. Malpresentation 5. Prolonged latent phase

2, 4, 5 Explanation: 2. Labor induction is a risk factor of dystocia. 4. Malpresentation is a risk factor of dystocia. 5. Prolonged latent phase is a risk factor of dystocia.

A client who has had a cesarean has been on bed rest for 8 hours after surgery and has warmth and redness in the left lower limb. Which interventions taken by the nurse would be most beneficial to the client? Select all that apply. 1. Advise the client to apply a hot compress at the reddened site 2. Inform the primary health care provider about the client's condition immediately 3. Advise the client to apply an antiinflammatory ointment at the reddened site 4. Have the client sit upright and lower the reddened leg 5. Have the client remain in bed with reddened limb elevated on pillows

2. Inform the primary health care provider about the client's condition immediately 5. Have the client remain in bed with reddened limb elevated on pillows

The nurse examines a woman 1 hour after birth. The woman's fundus is boggy, midline, and 1 cm below the umbilicus. Her lochial flow is profuse, with two plum-sized clots. What is the nurse's initial action? 1. Place her on a bedpan to empty her bladder 2. Massage her fundus 3. Call the physician 4. Administer methylergonovine (Methergine, 0.2 mg IM, which has been ordered prn)

2. Massage her fundus

The nurse observes that the perineal pad used by a postpartum client is saturated within 15 minutes, and there is blood pooling under the client's buttocks. What action does the nurse take? 1. Change the pad every 10 minutes. 2. Perform fundal massage. 3. Monitor the client's blood pressure. 4. Report the bleeding to the primary health care provider.

2. Perform fundal massage.

The nurse is caring for a client who delivered a baby girl 1 hour ago. The client is going into hypovolemic shock. What are the signs and symptoms she would exhibit? Select all that apply. 1. Skin feels warm and dry. 2. The woman's skin color turns ashen or grayish. 3. Pulse rate decreases and blood pressure increases. 4. The woman begins to act anxious or exhibits air hunger. 5. Persistent significant bleeding occurs—the perineal pad is soaked within 15 minutes. 6. The woman states she feels weak, lightheaded, "funny," nauseated, or that she "sees stars."

2. The woman's skin color turns ashen or grayish. 4. The woman begins to act anxious or exhibits air hunger. 5. Persistent significant bleeding occurs—the perineal pad is soaked within 15 minutes. 6. The woman states she feels weak, lightheaded, "funny," nauseated, or that she "sees stars."

Which statement accurately reflects the La cuarentena ritual for a Hispanic patient? 1. There are no restrictions placed on the mother during this ritual period. 2. This ritual occurs over a period of 40 days. 3. Spicy foods are encouraged as part of the maternal diet. 4. The ritual is limited to preparing the woman to become a good mother.

2. This ritual occurs over a period of 40 days.

A woman has been in labor for 16 hours. Her cervix is dilated to 3 cm and is 80% effaced. The fetal presenting part is not engaged. The nurse would suspect which of the following? 1. Breech malpresentation 2. Fetal demise 3. Cephalopelvic disproportion (CPD) 4. Abruptio placentae

3 Explanation: 3. Cephalopelvic disproportion (CPD) prevents the presenting part from becoming engaged.

The client vaginally delivers an infant that weighs 4750 g. Moderate shoulder dystocia occurred during the birth. During the initial assessment of this infant, what should the nurse look for? 1. Bells palsy 2. Bradycardia 3. Erb palsy 4. Petechiae

3 Explanation: 3. Macrosomic newborns should be inspected for cephalhematoma, Erb palsy, and fractured clavicles.

If the physician indicates a shoulder dystocia during the delivery of a macrosomic fetus, how would the nurse assist? 1. Call a second physician to assist. 2. Prepare for an immediate cesarean delivery. 3. Assist the woman into McRoberts maneuver. 4. Utilize fundal pressure to push the fetus out.

3 Explanation: 3. The McRoberts maneuver is thought to change the maternal pelvic angle and therefore reduce the force needed to extract the shoulders, thereby decreasing the incidence of brachial plexus stretching and clavicular fracture.

Lacerations of the cervix or vagina may be present when bright red vaginal bleeding persists in the presence of a well-contracted uterus. The incidence of lacerations is higher among which of the following childbearing women? Note: Credit will be given only if all correct choices and no incorrect choices are selected. Select all that apply. 1. Over the age of 35 2. Have not had epidural block 3. Have had an episiotomy 4. Have had a forceps-assisted or vacuum-assisted birth 5. Nulliparous

3, 4 Explanation: 3. The incidence of lacerations is higher among childbearing women who undergo an episiotomy. 4. The incidence of lacerations is higher among childbearing women who undergo forceps-assisted or vacuum-assisted birth.

True postterm pregnancies are frequently associated with placental changes that cause a decrease in the uterine-placental-fetal circulation. Complications related to alternations in placenta functioning include which of the following? Note: Credit will be given only if all correct choices and no incorrect choices are selected. Select all that apply. 1. Increased fetal oxygenation 2. Increased placental blood supply 3. Reduced nutritional supply 4. Macrosomia 5. Risk of shoulder dystocia

3, 4, 5 Explanation: 3. Reduced nutritional supply is a complication related to alternations in placenta functioning. 4. Macrosomia is a complication related to alternations in placenta functioning. 5. Risk of shoulder dystocia is a complication related to alternations in placenta functioning.

Five clients are in active labor in the labor unit. Which women should the nurse monitor carefully for the potential of uterine rupture? Note: Credit will be given only if all correct choices and no incorrect choices are selected. Select all that apply. 1. Age 15, in active labor 2. Age 22, with eclampsia 3. Age 25, last delivery by cesarean section 4. Age 32, first baby died during labor 5. Age 27, last delivery 11 months ago

3, 5 Explanation: 3. A woman who has had a previous cesarean section is at risk for uterine rupture. 5. A woman who does not have at least 18 months between deliveries is at greater risk for uterine rupture.

The nurse is assessing a postpartum client with uterine atony and pain in the lower abdomen. Upon interacting with the client, the nurse suspects that the client has a risk of fundal relaxation. Which statement by the client supports the nurse's assumption? 1. "I'm taking a warm sitz bath two times a day." 2. "I'm applying witch hazel compresses every day." 3. "I massage my uterus for 45 minutes every 8 hours." 4. "I apply an ice pack in the perineal area two times a day."

3. "I massage my uterus for 45 minutes every 8 hours."

A postpartum client who is taking analgesics for pain relief is anxious that the medication may pass into her breast milk and adversely affect the infant. What does the nurse tell the client? 1. "Medications do not pass into the breast milk." 2. "Take the medication just before bedtime." 3. "Take the medication immediately after breastfeeding." 4. "You need to avoid breastfeeding and use infant formula."

3. "Take the medication immediately after breastfeeding."

The nurse is preparing to assess the fundus of a postpartum patient. What nursing action is needed before assessment? 1. Have the patient to turn on her side. 2. Position the patient to lie flat on her back with her legs extended. 3. Ask the patient to urinate and empty her bladder. 4. Massage the fundus gently before determining its level.

3. Ask the patient to urinate and empty her bladder.

Which condition does the nurse assess in a postpartum client who does not breastfeed the newborn infant? 1. Sore nipples 2. Low estrogen levels 3. Breast engorgement 4. Postpartum depression

3. Breast engorgement

Excessive blood loss after childbirth can have several causes; however, which is the most common? 1. Vaginal or vulvar hematomas 2. Unrepaired lacerations of the vagina or cervix 3. Failure of the uterine muscle to contract firmly 4. Retained placental fragments

3. Failure of the uterine muscle to contract firmly

The primary health care provider suggests Kegel exercises to a postpartum client. A week later, the client complains of incontinence from the exercises. What does the nurse conclude from the client's condition? 1. The client has stopped breastfeeding the infant. 2. The client has not been performing the exercises at all. 3. The client has not performed the exercises correctly. 4. The client is experiencing a side effect of the exercises.

3. The client has not performed the exercises correctly.

The client has delivered a 4200 g fetus. The physician performed a midline episiotomy, which extended into a third-degree laceration. The client asks the nurse where she tore. Which response is best? 1. The episiotomy extended and tore through your rectal mucosa. 2. The episiotomy extended and tore up near your vaginal mucous membrane. 3. The episiotomy extended and tore into the muscle layer. 4. The episiotomy extended and tore through your anal sphincter.

4 Explanation: 4. A third degree laceration includes the anal sphincter.

The nurse is caring for a client in active labor. The membranes spontaneously rupture, with a large amount of clear amniotic fluid. Which nursing action is most important to undertake at this time? 1. Assess the odor of the amniotic fluid. 2. Perform Leopold maneuvers. 3. Obtain an order for pain medication. 4. Complete a sterile vaginal exam.

4 Explanation: 4. Checking the cervix will determine whether the cord prolapsed when the membranes ruptured. The nurse would assess for prolapsed cord via vaginal examination.

The nurse examines the clients placenta and finds that the umbilical cord is inserted at the placental margin. The client comments that the placenta and cord look different than they did for her first two births. The nurse should explain that this variation in placenta and cord is called what? 1. Placenta accreta 2. Circumvallate placenta 3. Succenturiate placenta 4. Battledore placenta

4 Explanation: 4. In battledore placenta, the umbilical cord is inserted at or near the placental margin.

The client is at 42 weeks gestation. Which order should the nurse question? 1. Obtain biophysical profile today. 2. Begin nonstress test now. 3. Schedule labor induction for tomorrow. 4. Have the client return to the clinic in 1 week.

4 Explanation: 4. Many practitioners use twice-weekly testing providing the amniotic fluid level is normal. One week is too long a period between assessments.

After delivery, it is determined that there is a placenta accreta. Which intervention should the nurse anticipate? 1. 2 L oxygen by mask 2. Intravenous antibiotics 3. Intravenous oxytocin 4. Hysterectomy

4 Explanation: 4. The primary complication of placenta accreta is maternal hemorrhage and failure of the placenta to separate following birth of the infant. An abdominal hysterectomy may be the necessary treatment, depending on the amount and depth of involvement.

The multiparous client at term has arrived to the labor and delivery unit in active labor with intact membranes. Leopold maneuvers indicate the fetus is in a transverse lie with a shoulder presentation. Which physician order is most important? 1. Artificially rupture membranes. 2. Apply internal fetal scalp electrode. 3. Monitor maternal blood pressure every 15 minutes. 4. Alert surgical team of urgent cesarean.

4 Explanation: 4. This is the highest priority because vaginal birth is impossible with a transverse lie. Labor should not be allowed to continue, and a cesarean birth is done quickly.

The nurse has taught perineal care techniques to a postpartum client to prevent infections. After the teaching session, the nurse asks the client to repeat the measures that should be followed to prevent infection. Which statement made by the client would indicate the need for further teaching? 1. "I should use soap and warm water to wash my perineum." 2. "I should wash from symphysis pubis back to anus." 3. "I should change the perineal pad for every 2 to 3 hours." 4. "I should fill the squeeze bottle with cold water while washing."

4. "I should fill the squeeze bottle with cold water while washing."

What intervention does the nurse perform to suppress lactation in a client who had a stillbirth? 1. Run warm water over the client's breasts. 2. Administer strong analgesics. 3. Administer oral and intravenous fluids. 4. Advise the client to wear a breast binder for the first 72 hours after giving birth.

4. Advise the client to wear a breast binder for the first 72 hours after giving birth.

The nurse observes that a postpartum client does not take pleasure in the infant and also fails to respond appropriately to infant cues. Which further assessment helps the nurse understand the client's condition? 1. Assessing the client's literacy 2. Monitoring the client's vital signs 3. Understanding the client's culture 4. Evaluating the client for postpartum depression

4. Evaluating the client for postpartum depression

On examining a woman who gave birth 5 hours ago, the nurse finds that the woman has completely saturated a perineal pad within 15 minutes. The nurse's first action is to do what? 1. Begin an IV infusion of Ringer's lactate solution 2. Assess the woman's vital signs 3. Call the woman's primary health care provider 4. Massage the woman's fundus

4. Massage the woman's fundus

Which finding would be a source of concern if noted during the assessment of a woman who is 12 hours postpartum? 1. Postural hypotension 2. Temperature of 38° C 3. Bradycardia-pulse rate of 55 beats/minute 4. Pain in left calf with dorsiflexion of left foot

4. Pain in left calf with dorsiflexion of left foot

The nurse can help a father in his transition to parenthood by: A. Pointing out that the infant turned at the sound of his voice B. Encouraging him to go home to get some sleep C. Telling him to tape the infant's diaper a different way D. Suggesting that he let the infant sleep in the bassinet

A A. Correct: Infants respond to the sound of voices. Because attachment involves a reciprocal interchange, observing the interaction between parent and infant is very important. B. Incorrect: Separation of the parent and infant does not encourage parent-infant attachment. C. Incorrect: Educating the parent in infant care techniques is important, but the manner in which a diaper is taped is not relevant and does not enhance parent-infant interactions. D. Incorrect: Parent-infant attachment involves touching, holding, and cuddling. It is appropriate for a father to want to hold the infant as the baby sleeps. p. 623

When the infant's behaviors and characteristics call forth a corresponding set of maternal behaviors and characteristics, this is called: A. Mutuality B. Bonding C. Claiming D. Acquaintance

A A. Correct: Mutuality extends the concept of attachment to include this shared set of behaviors. B. Incorrect: Bonding is the process over time of parents forming an emotional attachment to their infant. Mutuality refers to a shared set of behaviors that is a part of the bonding process. C. Incorrect: Claiming is the process by which parents identify their new baby in terms of likeness to other family members, their differences and uniqueness. Mutuality refers to a shared set of behaviors that is part of the bonding process. D. Incorrect: Like mutuality, acquaintance is part of attachment. It describes how parents get to know their baby during the immediate postpartum period through eye contact, touching, and talking. p. 613

The nurse notes that a Vietnamese woman does not cuddle or interact with her newborn other than to feed him, change his diapers or soiled clothes, and put him to bed. In evaluating the woman's behavior with her infant, the nurse realizes that: A. What appears to be a lack of interest in the newborn is in fact the Vietnamese way of demonstrating intense love by attempting to ward off evil spirits. B. The woman is inexperienced in caring for newborns. C. The woman needs a referral to a social worker for further evaluation of her parenting behaviors once she goes home with the newborn. D. Extra time needs to be planned for assisting the woman in bonding with her newborn.

A A. Correct: The nurse may observe a Vietnamese woman who gives minimal care to her infant and refuses to cuddle or interact with her infant. The apparent lack of interest in the newborn is this cultural group's attempt to ward off evil spirits and actually reflects an intense love and concern for the infant. B. Incorrect: Cultural beliefs are important determinates of parenting behaviors. The woman's "lack of interest" is a Vietnamese cultural behavior. C. Incorrect: Cultural beliefs are important determinates of parenting behaviors. The woman's "lack of interest" is a Vietnamese cultural behavior. The lack of infant interaction is not a form of infant neglect, but rather a demonstration of love and concern for the infant. D. Incorrect: The nurse may observe the woman and may be concerned by the apparent lack of interest in the newborn when, in fact, this is a cultural display of love and concern for the infant. It is important to educate the woman in infant care, but it is equally important to acknowledge her cultural beliefs and practices. p. 627

New parents express concern that because of the mother's emergency cesarean birth under general anesthesia, they did not have the opportunity to hold and bond with their daughter immediately after her birth. The nurse's response should convey to the parents that: A. Attachment, or bonding, is a process that occurs over time and does not require early contact. B. The time immediately after birth is a critical period for humans. C. Early contact is essential for optimum parent-infant relationships. D. They should just be happy that the infant is healthy.

A A. Correct: This statement is accurate. B. Incorrect: The formerly accepted definition of bonding held that the period immediately after birth was a critical time for bonding to occur. Research since has indicated that parent-infant attachment occurs over time. A delay does not inhibit the process. C. Incorrect: Parent-infant attachment involves activities such as touching, holding, and gazing; it is not exclusively eye contact. D. Incorrect: This response is inappropriate because it is derogatory and belittling. pp. 612-613

In the United States, the en face position is preferred immediately after birth. Nurses can facilitate this process by all of these actions except: A. Washing both the infant's face and the mother's face B. Placing the infant on the mother's abdomen or breast with their heads on the same plane C. Dimming the lights D. Delaying the instillation of prophylactic antibiotic ointment in the infant's eyes

A A. Correct: To facilitate the position in which the parent's and infant's faces are approximately 8 inches apart on the same plane, allowing them to make eye contact, the nurse can place the infant at the proper height on the mother's body, dim the light so that the infant's eyes open, and delay putting ointment in the infant's eyes. B. Incorrect: To facilitate the position in which the parent's and infant's faces are approximately 8 inches apart on the same plane, allowing them to make eye contact, the nurse can place the infant at the proper height on the mother's body, dim the light so that the infant's eyes open, and delay putting ointment in the infant's eyes. C. Incorrect: To facilitate the position in which the parent's and infant's faces are approximately 8 inches apart on the same plane, allowing them to make eye contact, the nurse can place the infant at the proper height on the mother's body, dim the light so that the infant's eyes open, and delay putting ointment in the infant's eyes. D. Incorrect: To facilitate the position in which the parent's and infant's faces are approximately 8 inches apart on the same plane, allowing them to make eye contact, the nurse can place the infant at the proper height on the mother's body, dim the light so that the infant's eyes open, and delay putting ointment in the infant's eyes. p. 617

Which condition is considered a medical emergency that requires immediate treatment? a. Inversion of the uterus b. Hypotonic uterus c. ITP d. Uterine atony

A Inversion of the uterus is likely to lead to hypovolemic shock and therefore is considered a medical emergency. Although hypotonic uterus, ITP, and uterine atony are serious conditions, they are not necessarily medical emergencies that require immediate treatment.

What is the most common reason for late postpartum hemorrhage (PPH)? a. Subinvolution of the uterus b. Defective vascularity of the decidua c. Cervical lacerations d. Coagulation disorders

A Late PPH may be the result of subinvolution of the uterus. Recognized causes of subinvolution include retained placental fragments and pelvic infection. Although defective vascularity, cervical lacerations, and coagulation disorders of the decidua may also cause PPH, late PPH typically results from subinvolution of the uterus, pelvic infection, or retained placental fragments.

A perinatal nurse is caring for a woman in the immediate postbirth period. Assessment reveals that the client is experiencing profuse bleeding. What is the most likely cause for this bleeding? a. Uterine atony b. Uterine inversion c. Vaginal hematoma d. Vaginal laceration

A Uterine atony is significant hypotonia of the uterus and is the leading cause of postpartum hemorrhage. Uterine inversion may lead to hemorrhage; however, it is not the most likely source of this clients bleeding. Further, if the woman were experiencing a uterine inversion, it would be evidenced by the presence of a large, red, rounded mass protruding from the introitus. A vaginal hematoma may be associated with hemorrhage. However, the most likely clinical finding for vaginal hematoma is pain, not the presence of profuse bleeding. A vaginal laceration should be suspected if vaginal bleeding continues in the presence of a firm, contracted uterine fundus.

Which medications are used to manage PPH? (Select all that apply.) a. Oxytocin b. Methergine c. Terbutaline d. Hemabate e. Magnesium sulfate

A, B, D Oxytocin, Methergine, and Hemabate are medications used to manage PPH. Terbutaline and magnesium sulfate are tocolytic medications that are used to relax the uterus, which would cause or worsen PPH.

What concerns about parenthood are often expressed by visually impaired mothers? Choose all that apply. A. Infant safety B. Transportation C. The ability to care for the infant D. Missing out visually E. Needing extra time for parenting activities to accommodate the visual limitations

A, B, D, E Correct: Concerns expressed by visually impaired mothers include infant safety, extra time needed for parenting activities, transportation, handling other people's reactions, providing proper discipline, and missing out visually. Incorrect: Blind people sense reluctance on the part of others to acknowledge that they have a right to be parents. However, blind parents are fully capable of caring for their infants. p. 628

Lacerations of the cervix, vagina, or perineum are also causes of PPH. Which factors influence the causes and incidence of obstetric lacerations of the lower genital tract? (Select all that apply.) a. Operative and precipitate births b. Adherent retained placenta c. Abnormal presentation of the fetus d. Congenital abnormalities of the maternal soft tissue e. Previous scarring from infection

A, C, D, E Abnormal adherence of the placenta occurs for unknown reasons. Attempts to remove the placenta in the usual manner can be unsuccessful, and lacerations or a perforation of the uterine wall may result. However, attempts to remove the placenta do not influence lower genital tract lacerations. Lacerations of the perineum are the most common of all lower genital tract injuries and often occur with both precipitate and operative births and are classified as first-, second-, third-, and fourth-degree lacerations. An abnormal presentation or position of the fetus, the relative size of the presenting part, and the birth canal may contribute to lacerations of the lower genital tract. Congenital abnormalities, previous scarring from infection or injury, and a contracted pelvis may also influence injury to the lower genital tract, followed by hemorrhage.

23. A client at 39 weeks of gestation has been admitted for an external version. Which intervention would the nurse anticipate the provider to order? a. Tocolytic drug b. Contraction stress test (CST) c. Local anesthetic d. Foley catheter

ANS: A A tocolytic drug will relax the uterus before and during the version, thus making manipulation easier. CST is used to determine the fetal response to stress. A local anesthetic is not used with external version. Although the bladder should be emptied, catheterization is not necessary. DIF: Cognitive Level: Apply REF: p. 779 TOP: Nursing Process: Planning MSC: Client Needs: Physiologic Integrity

8. A woman gave birth to a 7-lb, 6-oz infant girl 1 hour ago. The birth was vaginal and the estimated blood loss (EBL) was 1500 ml. When assessing the woman's vital signs, the nurse is concerned to see: a. Temperature 37.9° C, heart rate 120, respirations 20, blood pressure 90/50 b. Temperature 37.4° C, heart rate 88, respirations 36, blood pressure 126/68 c. Temperature 38° C, heart rate 80, respirations 16, blood pressure 110/80 d. Temperature 36.8° C, heart rate 60, respirations 18, blood pressure 140/90

ANS: A An EBL of 1500 ml with tachycardia and hypotension suggests hypovolemia caused by excessive blood loss. Temperature 37.4° C, heart rate 88, respirations 36, blood pressure 126/68 are normal vital signs except for an increased respiratory rate, which may be secondary to pain from the birth. Temperature 38° C, heart rate 80, respirations 16, blood pressure 110/80 are normal vital signs except for the temperature. However, temperature may increase to 38° C during the first 24 hours as a result of the dehydrating effects of labor. Temperature 36.8° C, heart rate 60, respirations 18, blood pressure 140/90 are normal vital signs, although the blood pressure is slightly elevated, which may be due to the use of oxytocic medications.

17. Rho immune globulin will be ordered postpartum if which situation occurs? a. Mother Rh, baby Rh+ b. Mother Rh, baby Rh c. Mother Rh+, baby Rh+ d. Mother Rh+, baby Rh

ANS: A An Rh mother delivering an Rh+ baby may develop antibodies to fetal cells that entered her bloodstream when the placenta separated. The Rho immune globulin works to destroy the fetal cells in the maternal circulation before sensitization occurs. If mother and baby are both Rh+ or Rh the blood types are alike, so no antibody formation would be anticipated. If the Rh+ blood of the mother comes in contact with the Rh blood of the infant, no antibodies would develop because the antigens are in the mothers blood, not in the infants.

4. A woman in preterm labor at 30 weeks of gestation receives two 12-mg intramuscular (IM) doses of betamethasone. What is the purpose of this pharmacologic intervention? a. To stimulate fetal surfactant production b. To reduce maternal and fetal tachycardia associated with ritodrine administration c. To suppress uterine contractions d. To maintain adequate maternal respiratory effort and ventilation during magnesium sulfate therapy

ANS: A Antenatal glucocorticoids administered as IM injections to the mother accelerate fetal lung maturity. Propranolol (Inderal) is given to reduce the effects of ritodrine administration. Betamethasone has no effect on uterine contractions. Calcium gluconate is given to reverse the respiratory depressive effects of magnesium sulfate therapy. DIF: Cognitive Level: Understand REF: p. 769 TOP: Nursing Process: Planning MSC: Client Needs: Physiologic Integrity

New parents express concern that, because of the mother's emergency cesarean birth under general anesthesia, they did not have the opportunity to hold and bond with their daughter immediately after her birth. The nurse's response should convey to the parents that: a.Attachment, or bonding, is a process that occurs over time and does not require early contact. b.The time immediately after birth is a critical period for people. c.Early contact is essential for optimum parent-infant relationships. d.They should just be happy that the infant is healthy.

ANS: A Attachment, or bonding, is a process that occurs over time and does not require early contact. The formerly accepted definition of bonding held that the period immediately after birth was a critical time for bonding to occur. Research since has indicated that parent-infant attachment occurs over time. A delay does not inhibit the process. Parent-infant attachment involves activities such as touching, holding, and gazing; it is not exclusively eye contact. A response that conveys that the parents should just be happy that the infant is healthy is inappropriate because it is derogatory and belittling.

15. Which description most accurately describes the augmentation of labor? a. Is part of the active management of labor that is instituted when the labor process is unsatisfactory b. Relies on more invasive methods when oxytocin and amniotomy have failed c. Is a modern management term to cover up the negative connotations of forceps-assisted birth d. Uses vacuum cups

ANS: A Augmentation is part of the active management of labor that stimulates uterine contractions after labor has started but is not progressing satisfactorily. Augmentation uses amniotomy and oxytocin infusion, as well as some more gentle, noninvasive methods. Forceps-assisted births are less common than in the past and not considered a method of augmentation. A vacuum-assisted delivery occurs during childbirth if the mother is too exhausted to push. Vacuum extraction is not considered an augmentation methodology. DIF: Cognitive Level: Understand REF: pp. 785-786 TOP: Nursing Process: Planning MSC: Client Needs: Health Promotion and Maintenance

10. The self-destruction of excess hypertrophied tissue in the uterus is called: a. Autolysis b. Subinvolution c. Afterpain d. Diastasis

ANS: A Autolysis is caused by a decrease in hormone levels. Subinvolution is failure of the uterus to return to a nonpregnant state. Afterpain is caused by uterine cramps 2 to 3 days after birth. Diastasis refers to the separation of muscles.

13. Knowing that the condition of the new mother's breasts will be affected by whether she is breastfeeding, nurses should be able to tell their clients all of these statements except: a. Breast tenderness is likely to persist for about a week after the start of lactation b. As lactation is established, a mass may form that can be distinguished from cancer by its position shift from day to day c. In nonlactating mothers, colostrum is present for the first few days after childbirth d. If suckling is never begun (or is discontinued), lactation ceases within a few days to a week

ANS: A Breasts become fuller and heavier as colostrum transitions to milk. This should last 72 to 96 hours. The movable, noncancerous mass is a filled milk sac. Colostrum is present for a few days whether the mother breastfeeds or not. A mother who does not want to breastfeed should also avoid stimulating her nipples.

15. When should discharge instruction, or the teaching plan that tells the woman what she needs to know to care for herself and her newborn, officially begin? a. At the time of admission to the nurses unit b. When the infant is presented to the mother at birth c. During the first visit with the physician in the unit d. When the take-home information packet is given to the couple

ANS: A Discharge planning, the teaching of maternal and newborn care, begins on the womans admission to the unit, continues throughout her stay, and actually never ends as long as she has contact with medical personnel.

The mother-baby nurse is able to recognize reciprocal attachment behavior. This refers to: a.The positive feedback an infant exhibits toward parents during the attachment process. b.Behavior during the sensitive period when the infant is in the quiet alert stage. c.Unidirectional behavior exhibited by the infant, initiated and enhanced by eye contact. d.Behavior by the infant during the sensitive period to elicit feelings of "falling in love" from the parents.

ANS: A In this definition, "reciprocal" refers to the feedback from the infant during the attachment process. This is a good time for bonding; however, it does not define reciprocal attachment. Reciprocal attachment applies to feedback behavior and is not unidirectional.

14. In many hospitals, new mothers are routinely presented with gift bags containing samples of infant formula. This practice is inconsistent with what? a. Baby Friendly Hospital Initiative b. Promotion of longer periods of breastfeeding c. Perception of being supportive to both bottle feeding and breastfeeding mothers d. Association with earlier cessation of breastfeeding

ANS: A Infant formula should not be given to mothers who are breastfeeding. Such gifts are associated with early cessation of breastfeeding. Baby Friendly USA prohibits the distribution of any gift bags or formula to new mothers.

The nurse can help a father in his transition to parenthood by: a.Pointing out that the infant turned at the sound of his voice. b.Encouraging him to go home to get some sleep. c.Telling him to tape the infant's diaper a different way. d.Suggesting that he let the infant sleep in the bassinet.

ANS: A Infants respond to the sound of voices. Because attachment involves a reciprocal interchange, observing the interaction between parent and infant is very important. Separation of the parent and infant does not encourage parent-infant attachment. Educating the parent in infant care techniques is important; however, the manner in which a diaper is taped is not relevant and does not enhance parent-infant interactions. Parent-infant attachment involves touching, holding, and cuddling. It is appropriate for a father to want to hold the infant as the baby sleeps.

16. The exact cause of preterm labor is unknown but believed to be multifactorial. Infection is thought to be a major factor in many preterm labors. Which type of infection has not been linked to preterm birth? a. Viral b. Periodontal c. Cervical d. Urinary tract

ANS: A Infections that increase the risk of preterm labor and birth are bacterial and include cervical, urinary tract, periodontal, and other bacterial infections. Therefore, early, continual, and comprehensive participation by the client in her prenatal care is important. Recent evidence has shown a link between periodontal infections and preterm labor. Researchers recommend regular dental care before and during pregnancy, oral assessment as a routine part of prenatal care, and scrupulous oral hygiene to prevent periodontal infections. DIF: Cognitive Level: Remember REF: p. 760 TOP: Nursing Process: Assessment MSC: Client Needs: Physiologic Integrity

When the infant's behaviors and characteristics call forth a corresponding set of maternal behaviors and characteristics, this is called: a.Mutuality. c.Claiming. b.Bonding. d.Acquaintance.

ANS: A Mutuality extends the concept of attachment to include this shared set of behaviors. Bonding is the process over time of parents forming an emotional attachment to their infant. Mutuality refers to a shared set of behaviors that is a part of the bonding process. Claiming is the process by which parents identify their new baby in terms of likeness to other family members and their differences and uniqueness. Like mutuality, acquaintance is part of attachment. It describes how parents get to know their baby during the immediate postpartum period through eye contact, touching, and talking.

8. A hospital has a number of different perineal pads available for use. A nurse is observed soaking several of them and writing down what she sees. What goal is the nurse attempting to achieve by performing this practice? a. To improve the accuracy of blood loss estimation, which usually is a subjective assessment b. To determine which pad is best c. To demonstrate that other nurses usually underestimate blood loss d. To reveal to the nurse supervisor that one of them needs some time off

ANS: A Saturation of perineal pads is a critical indicator of excessive blood loss; anything done to help in the assessment is valuable. The nurse is noting the saturation volumes and soaking appearances. Instead of determining which pad is best, the nurse is more likely noting saturation volumes and soaking appearances to improve the accuracy of estimated blood loss. Nurses usually overestimate blood loss. Soaking perineal pads and writing down the results does not indicate the need for time off of work.

5. A primiparous woman is to be discharged from the hospital the following day with her infant girl. Which behavior indicates a need for further intervention by the nurse before the woman can be discharged? a. The woman is disinterested in learning about infant care. b. The woman continues to hold and cuddle her infant after she has fed her. c. The woman reads a magazine while her infant sleeps. d. The woman changes her infants diaper and then shows the nurse the contents of the diaper.

ANS: A The client should be excited, happy, and interested or involved in infant care. A woman who is sad, tearful, or disinterested in caring for her infant may be exhibiting signs of depression or postpartum blues and may require further intervention. Holding and cuddling her infant after feeding is an appropriate parent-infant interaction. Taking time for herself while the infant is sleeping is an appropriate maternal action. Showing the nurse the contents of the diaper is appropriate because the mother is seeking approval from the nurse and notifying the nurse of the infants elimination patterns.

25. Which nursing intervention should be immediately performed after the forceps-assisted birth of an infant? a. Assessing the infant for signs of trauma b. Administering prophylactic antibiotic agents to the infant c. Applying a cold pack to the infant's scalp d. Measuring the circumference of the infant's head

ANS: A The infant should be assessed for bruising or abrasions at the site of application, facial palsy, and subdural hematoma. Prophylactic antibiotics are not necessary with a forceps delivery. A cold pack would place the infant at risk for cold stress and is contraindicated. Measuring the circumference of the head is part of the initial nursing assessment. DIF: Cognitive Level: Apply REF: p. 788 TOP: Nursing Process: Implementation MSC: Client Needs: Physiologic Integrity

16. Several changes in the integumentary system that appear during pregnancy disappear after birth, although not always completely. What change is almost certain to be completely reversed? a. Nail brittleness b. Darker pigmentation of the areolae and linea nigra c. Striae gravidarum on the breasts, abdomen, and thighs d. Spider nevi

ANS: A The nails return to their prepregnancy consistency and strength. Some women have permanent darker pigmentation of the areolae and linea nigra. However, the nails return to their prepregnancy consistency and strength. Striae gravidarum (stretch marks) usually do not completely disappear. However, the nails return to their prepregnancy consistency and strength. For some women, spider nevi persist indefinitely. However, the nails return to their prepregnancy consistency and strength.

The nurse notes that a Vietnamese woman does not cuddle or interact with her newborn other than to feed him, change his diapers or soiled clothes, and put him to bed. In evaluating the woman's behavior with her infant, the nurse realizes that: a.What appears to be a lack of interest in the newborn is in fact the Vietnamese way of demonstrating intense love by attempting to ward off evil spirits. b.The woman is inexperienced in caring for newborns. c.The woman needs a referral to a social worker for further evaluation of her parenting behaviors once she goes home with the newborn. d.Extra time needs to be planned for assisting the woman in bonding with her newborn.

ANS: A The nurse may observe a Vietnamese woman who gives minimal care to her infant and refuses to cuddle or interact with her infant. The apparent lack of interest in the newborn is this cultural group's attempt to ward off evil spirits and actually reflects an intense love and concern for the infant. It is important to educate the woman in infant care, but it is equally important to acknowledge her cultural beliefs and practices.

20. A pregnant woman's amniotic membranes have ruptured. A prolapsed umbilical cord is suspected. What intervention would be the nurse's highest priority? a. Placing the woman in the knee-chest position b. Covering the cord in sterile gauze soaked in saline c. Preparing the woman for a cesarean birth d. Starting oxygen by face mask

ANS: A The woman is assisted into a modified Sims position, Trendelenburg position, or the knee-chest position in which gravity keeps the pressure of the presenting part off the cord. Although covering the cord in sterile gauze soaked saline, preparing the woman for a cesarean, and starting oxygen by face mark are appropriate nursing interventions in the event of a prolapsed cord, the intervention of top priority would be positioning the mother to relieve cord compression. DIF: Cognitive Level: Apply REF: p. 797 TOP: Nursing Process: Implementation MSC: Client Needs: Physiologic Integrity

2. The laboratory results for a postpartum woman are as follows: blood type, A; Rh status, positive; rubella titer, 1:8 (enzyme immunoassay [EIA] 0.8); hematocrit, 30%. How should the nurse best interpret these data? a.Rubella vaccine should be administered. b.Blood transfusion is necessary. c.Rh immune globulin is necessary within 72 hours of childbirth. d. Kleihauer-Betke test should be performed.

ANS: A This clients rubella titer indicates that she is not immune and needs to receive a vaccine. These data do not indicate that the client needs a blood transfusion. Rh immune globulin is indicated only if the client has an Rh-negative status and the infant has an Rh-positive status. A Kleihauer-Betke test should be performed if a large fetomaternal transfusion is suspected, especially if the mother is Rh negative. However, the data provided do not indicate a need for performing this test.

In the United States the en face position is preferred immediately after birth. Nurses can facilitate this process by all of these actions except: a.Washing both the infant's face and the mother's face. b.Placing the infant on the mother's abdomen or breast with their heads on the same plane. c.Dimming the lights. d.Delaying the instillation of prophylactic antibiotic ointment in the infant's eyes.

ANS: A To facilitate the position in which the parent's and infant's faces are approximately 8 inches apart on the same plane, allowing them to make eye contact, the nurse can place the infant at the proper height on the mother's body, dim the light so that the infant's eyes open, and delay putting ointment in the infant's eyes.

1. A woman gave birth to an infant boy 10 hours ago. Where does the nurse expect to locate this woman's fundus? a. At the level of the umbilicus b. Two centimeters below the umbilicus c. Midway between the umbilicus and the symphysis pubis d. Nonpalpable abdominally

ANS: A Within 12 hours after delivery the fundus rises to approximately the level of the umbilicus. The fundus descends about 1 to 2 cm every 24 hours. Within 12 hours after delivery the fundus may be approximately 1 cm above the umbilicus. By the sixth postpartum week the fundus normally is halfway between the symphysis pubis and the umbilicus. The fundus should be easily palpated using the maternal umbilicus as a reference point.

2. What are the complications and risks associated with cesarean births? (Select all that apply.) a. Pulmonary edema b. Wound dehiscence c. Hemorrhage d. Urinary tract infections e. Fetal injuries

ANS: A, B, C, D, E Pulmonary edema, wound dehiscence, hemorrhage, urinary tract infections, and fetal injuries are possible complications and risks associated with cesarean births. DIF: Cognitive Level: Understand REF: p. 790 TOP: Nursing Process: Evaluation MSC: Client Needs: Physiologic Integrity

3. Women who are obese are at risk for several complications during pregnancy and birth. Which of these would the nurse anticipate with an obese client? (Select all that apply.) a. Thromboembolism b. Cesarean birth c. Wound infection d. Breech presentation e. Hypertension

ANS: A, B, C, E A breech presentation is not a complication of pregnancy or birth for the client who is obese. Venous thromboembolism is a known risk for obese women. Therefore, the use of thromboembolism-deterrent (TED) hose and sequential compression devices may help decrease the chance for clot formation. Women should also be encouraged to ambulate as soon as possible. In addition to having an increased risk for complications with a cesarean birth, in general, obese women are also more likely to require an emergency cesarean birth. Many obese women have a pannus (i.e., large roll of abdominal fat) that overlies a lower transverse incision made just above the pubic area. The pannus causes the area to remain moist, which encourages the development of infection. Obese women are more likely to begin pregnancy with comorbidities such as hypertension and type 2 diabetes. DIF: Cognitive Level: Analyze REF: p. 778 TOP: Nursing Process: Assessment MSC: Client Needs: Health Promotion and Maintenance

4. If a woman is at risk for thrombus and is not ready to ambulate, which nursing intervention would the nurse use? (Select all that apply.) a. Putting her in antiembolic stockings (thromboembolic deterrent [TED] hose) and/or sequential compression device (SCD) boots b. Having her flex, extend, and rotate her feet, ankles, and legs c. Having her sit in a chair d. Immediately notifying the physician if a positive Homans sign occurs e. Promoting bed rest

ANS: A, B, D Sitting immobile in a chair does not help; bed exercise and prophylactic footwear might. TED hose and SCD boots are recommended. The client should be encouraged to ambulate with assistance, not remain in bed. Bed exercises are useful. A positive Homans sign (calf muscle pain or warmth, redness, tenderness) requires the physicians immediate attention.

3. Which physiologic factors are reliable indicators of impending shock from postpartum hemorrhage? (Select all that apply.) a. Respirations b. Skin condition c. Blood pressure d. Level of consciousness e. Urinary output

ANS: A, B, D, E Blood pressure is not a reliable indicator; several more sensitive signs are available. Blood pressure does not drop until 30% to 40% of blood volume is lost. Respirations, pulse, skin condition, urinary output, and level of consciousness are more sensitive means of identifying hypovolemic shock.

Which concerns about parenthood are often expressed by visually impaired mothers (Select all that apply)? a.Infant safety b.Transportation c.The ability to care for the infant d.Missing out visually e.Needing extra time for parenting activities to accommodate the visual limitations

ANS: A, B, D, E Concerns expressed by visually impaired mothers include infant safety, extra time needed for parenting activities, transportation, handling other people's reactions, providing proper discipline, and missing out visually. Blind people sense reluctance on the part of others to acknowledge that they have a right to be parents; however, blind parents are fully capable of caring for their infants.

2. Which practices contribute to the prevention of postpartum infection? (Select all that apply.) a. Not allowing the mother to walk barefoot at the hospital b. Educating the client to wipe from back to front after voiding c. Having staff members with conditions such as strep throat, conjunctivitis, and diarrhea stay home d. Instructing the mother to change her perineal pad from front to back each time she voids or defecates e. Not permitting visitors with cough or colds to enter the postpartum unit

ANS: A, C, D Proper perineal care helps prevent infection and aids in the healing process. Educating the woman to wipe from front to back (urethra to anus) after voiding or defecating is a simple first step. Walking barefoot and getting back into bed can contaminate the linens. Clients should wear shoes or slippers. Staff members with infections need to stay home until they are no longer contagious. The client should also wash her hands before and after these functions. Visitors with any signs of illness should not be allowed entry to the postpartum unit.

4. The induction of labor is considered an acceptable obstetric procedure if it is in the best interest to deliver the fetus. The charge nurse on the labor and delivery unit is often asked to schedule clients for this procedure and therefore must be cognizant of the specific conditions appropriate for labor induction. What are appropriate indications for induction? (Select all that apply?) a. Rupture of membranes at or near term b. Convenience of the woman or her physician c. Chorioamnionitis (inflammation of the amniotic sac) d. Postterm pregnancy e. Fetal death

ANS: A, C, D, E The conditions listed are all acceptable indications for induction. Other conditions include intrauterine growth restriction (IUGR), maternal-fetal blood incompatibility, hypertension, and placental abruption. Elective inductions for the convenience of the woman or her provider are not recommended; however, they have become commonplace. Factors such as rapid labors and living a long distance from a health care facility may be valid reasons in such a circumstance. Elective delivery should not occur before 39 weeks of completed gestation. DIF: Cognitive Level: Apply REF: p. 790 TOP: Nursing Process: Planning MSC: Client Needs: Physiologic Integrity

1. Many new mothers experience some type of nipple pain during the first weeks of initiating breastfeeding. Should this pain be severe or persistent, it may discourage or inhibit breastfeeding altogether. Which factors might contribute to this pain? (Select all that apply.) a. Improper feeding position b. Large-for-gestational age infant c. Fair skin d. Progesterone deficiency e. Flat or retracted nipples

ANS: A, C, E Nipple lesions may manifest as chapped, cracked, bleeding, sore, erythematous, edematous, or blistered nipples. Factors that contribute to nipple pain include improper positioning or a failure to break suction before removing the baby from the breast. Flat or retracted nipples along with the use of nipple shields, breast shells, or plastic breast pads also contribute to nipple pain. Women with fair skin are more likely to develop sore and cracked nipples. Preventing nipple soreness is preferable to treating soreness after it appears. Vigorous feeding may be a contributing factor, which may be the case with any size infant, not just infants who are large for gestational age. Estrogen or dietary deficiencies can contribute to nipple soreness.

6. Nurses play a critical role in educating parents regarding measures to prevent infant abduction. Which instructions contribute to infant safety and security? (Select all that apply.) a. The mother should check the photo identification (ID) of any person who comes to her room. b. The baby should be carried in the parents arms from the room to the nursery. c. Because of infant security systems, the baby can be left unattended in the clients room. d. Parents should use caution when posting photographs of their infant on the Internet. e. The mom should request that a second staff member verify the identity of any questionable person.

ANS: A, D, E Nurses must discuss infant security precautions with the mother and her family because infant abduction continues to be a concern. The mother should be taught to check the identity of any person who comes to remove the baby from her room. Hospital personnel usually wear picture identification patches. On some units, staff members also wear matching scrubs or special badges that are unique to the perinatal unit. As a rule, the baby is never carried in arms between the mothers room and the nursery, but rather the infant is always wheeled in a bassinet. The infant should never be left unattended, even if the facility has an infant security system. Parents should be instructed to use caution when posting photographs of their new baby on the Internet and on other public forums.

A parent who has a hearing impairment is presented with a number of challenges in parenting. Which nursing approaches are appropriate for working with hearing-impaired new parents (Select all that apply)? a.Use devices that transform sound into light. b.Assume that the patient knows sign language. c.Speak quickly and loudly. d.Ascertain whether the patient can read lips before teaching. e.Written messages aid in communication.

ANS: A, D, E Section 504 of the Rehabilitation Act of 1973 requires that hospitals use various communication techniques and resources with the deaf and hard of hearing patient. This includes devices such as door alarms, cry alarms, and amplifiers. Before initiating communication, the nurse needs to be aware of the parents' preferences for communication. Not all hearing-impaired patients know sign language. Do they wear a hearing aid? Do they read lips? Do they wish to have a sign language interpreter? If the parent relies on lip reading, the nurse should sit close enough so that the parent can visualize lip movements. The nurse should speak clearly in a regular voice volume, in short, simple sentences. Written messages such as on a black or white erasable board can be useful. Written materials should be reviewed with the parents before discharge.

Many first-time parents do not plan on their parents' help immediately after the newborn arrives. What statement by the nurse is the most appropriate when counseling new parents about the involvement of grandparents? a."You should tell your parents to leave you alone." b."Grandparents can help you with parenting skills and also help preserve family traditions." c."Grandparent involvement can be very disruptive to the family." d."They are getting old. You should let them be involved while they can."

ANS: B "Grandparents can help you with parenting skills and also help preserve family traditions" is the most appropriate response. Intergenerational help may be perceived as interference; however, a statement of this sort is not therapeutic to the adaptation of the family. Not only is "Grandparent involvement can be very disruptive to the family" invalid, it also is not an appropriate nursing response. Regardless of age, grandparents can help with parenting skills and preserve family traditions. Talking about the age of the grandparents is not the most appropriate statement, and it does not demonstrate sensitivity on the part of the nurse.

24. What is a maternal indication for the use of vacuum-assisted birth? a. Wide pelvic outlet b. Maternal exhaustion c. History of rapid deliveries d. Failure to progress past station 0

ANS: B A mother who is exhausted may be unable to assist with the expulsion of the fetus. The client with a wide pelvic outlet will likely not require vacuum extraction. With a rapid delivery, vacuum extraction is not necessary. A station of 0 is too high for a vacuum-assisted birth. DIF: Cognitive Level: Understand REF: p. 786 TOP: Nursing Process: Assessment MSC: Client Needs: Physiologic Integrity

19. When caring for a newly delivered woman, what is the best measure to prevent abdominal distention after a cesarean birth? a. Rectal suppositories b. Early and frequent ambulation c. Tightening and relaxing abdominal muscles d. Carbonated beverages

ANS: B Activity will aid the movement of accumulated gas in the gastrointestinal tract. Rectal suppositories can be helpful after distention occurs; however, they do not prevent it. Ambulation is the best prevention. Carbonated beverages may increase distention.

3. Which woman is most likely to experience strong afterpains? a. A woman who experienced oligohydramnios b. A woman who is a gravida 4, para 4-0-0-4 c. A woman who is bottle feeding her infant d. A woman whose infant weighed 5 lb, 3 oz

ANS: B Afterpains are more common in multiparous women. In a woman who experienced polyhydramnios, afterpains are more noticeable because the uterus was greatly distended. Breastfeeding may cause afterpains to intensify. In a woman who delivered a large infant, afterpains are more noticeable because the uterus was greatly distended.

3. A woman gave birth 48 hours ago to a healthy infant girl. She has decided to bottle feed. During the assessment, the nurse notices that both breasts are swollen, warm, and tender on palpation. Which guidance should the nurse provide to the client at this time? a. Run warm water on her breasts during a shower. b. Apply ice to the breasts for comfort. c. Express small amounts of milk from the breasts to relieve the pressure. d. Wearing a loose-fitting bra to prevent nipple irritation.

ANS: B Applying ice packs and cabbage leaves to the breasts for comfort is an appropriate intervention for treating engorgement in a mother who is bottle feeding. The ice packs should be applied for 15 minutes on and 45 minutes off to avoid rebound engorgement. A bottle-feeding mother should avoid any breast stimulation, including pumping or expressing milk. A bottle-feeding mother should continuously wear a well-fitted support bra or breast binder for at least the first 72 hours after giving birth. A loose-fitting bra will not aid lactation suppression. Furthermore, the shifting of the bra against the breasts may stimulate the nipples and thereby stimulate lactation.

To promote bonding and attachment immediately after delivery, the most important nursing intervention is to: a.Allow the mother quiet time with her infant. b.Assist the mother in assuming an en face position with her newborn. c.Teach the mother about the concepts of bonding and attachment. d.Assist the mother in feeding her baby.

ANS: B Assisting the mother in assuming an en face position with her newborn will support the bonding process. The mother should be given as much privacy as possible; however, nursing assessments must still be continued during this critical time. The mother has just delivered and is more focused on the infant; she will not be receptive to teaching at this time. This is a good time to initiate breastfeeding; however, the mother first needs time to explore the new infant and begin the bonding process.

14. A number of methods can be used for inducing labor. Which cervical ripening method falls under the category of mechanical or physical? a. Prostaglandins are used to soften and thin the cervix. b. Labor can sometimes be induced with balloon catheters or laminaria tents. c. Oxytocin is less expensive and more effective than prostaglandins but creates greater health risks. d. Amniotomy can be used to make the cervix more favorable for labor.

ANS: B Balloon catheters or laminaria tents are mechanical means of ripening the cervix. Ripening the cervix, making it softer and thinner, increases the success rate of induced labor. Prostaglandin E1 is less expensive and more effective than oxytocin but carries a greater risk. Amniotomy is the artificial rupture of membranes, which is used to induce labor only when the cervix is already ripe. DIF: Cognitive Level: Apply REF: p. 781 TOP: Nursing Process: Planning MSC: Client Needs: Health Promotion and Maintenance

10. The nurse who elects to work in the specialty of obstetric care must have the ability to distinguish between preterm birth, preterm labor, and low birth weight. Which statement regarding this terminology is correct? a. Terms preterm birth and low birth weight can be used interchangeably. b. Preterm labor is defined as cervical changes and uterine contractions occurring between 20 and 37 weeks of gestation. c. Low birth weight is a newborn who weighs below 3.7 pounds. d. Preterm birth rate in the United States continues to increase.

ANS: B Before 20 weeks of gestation, the fetus is not viable (miscarriage); after 37 weeks, the fetus can be considered term. Although these terms are used interchangeably, they have different meanings: preterm birth describes the length of gestation (before 37 weeks), regardless of the newborn's weight; low birth weight describes only the infant's weight at the time of birth (2500 g or less), whenever it occurs. Low birth weight is anything below 2500 g or approximately pounds. In 2011, the preterm birth rate in the United States was 11.7 %; it has dropped every year since 2008. DIF: Cognitive Level: Understand REF: p. 759 TOP: Nursing Process: Assessment MSC: Client Needs: Health Promotion and Maintenance

A man calls the nurse's station and states that his wife, who delivered 2 days ago, is happy one minute and crying the next. The man says, "She was never like this before the baby was born." The nurse's initial response could be to: a.Tell him to ignore the mood swings, as they will go away. b.Reassure him that this behavior is normal. c.Advise him to get immediate psychological help for her. d.Instruct him in the signs, symptoms, and duration of postpartum blues.

ANS: B Before providing further instructions, inform family members of the fact that postpartum blues are a normal process. Telling her partner to "ignore the mood swings" does not encourage further communication and may belittle the husband's concerns. Postpartum blues are usually short-lived; no medical intervention is needed. Client teaching is important; however, the new father's anxieties need to be allayed before he will be receptive to teaching.

In follow-up appointments or visits with parents and their new baby, it may be useful if the nurse can identify parental behaviors that can either facilitate or inhibit attachment. Which one is a facilitating behavior? a.The parents have difficulty naming the infant. b.The parents hover around the infant, directing attention to and pointing at the infant. c.The parents make no effort to interpret the actions or needs of the infant. d.The parents do not move from fingertip touch to palmar contact and holding.

ANS: B Hovering over the infant and obviously paying attention to the baby are facilitating behaviors. Inhibiting behaviors include difficulty naming the infant, making no effort to interpret the actions or needs of the infant, and not moving from fingertip touch to palmar contact and holding.

11. With regard to the postpartum uterus, nurses should be aware that: a. At the end of the third stage of labor, it weighs approximately 500 g b. After 2 weeks postpartum, it should not be palpable abdominally c. After 2 weeks postpartum, it weighs 100 g d. It returns to its original (prepregnancy) size by 6 weeks postpartum

ANS: B However, the uterus does not return to its original size. At the end of the third stage of labor, the uterus weighs approximately 1000 g. It does not return to its original size. After 2 weeks postpartum the uterus weighs about 350 g. It does not return to its original size. The normal self-destruction of excess hypertrophied tissue accounts for the slight increase in uterine size after each pregnancy.

Other early sensual contacts between infant and mother involve sound and smell. Nurses should be aware that, despite what folk wisdom may say: a.High-pitched voices irritate newborns. b.Infants can learn to distinguish their mother's voice from others soon after birth. c.All babies in the hospital smell alike. d.A mother's breast milk has no distinctive odor.

ANS: B Infants know the sound of their mother's voice early. Infants respond positively to high-pitched voices. Each infant has a unique odor. Infants quickly learn to distinguish the odor of their mother's breast milk.

When working with parents who have some form of sensory impairment, nurses should understand that ________ is an inaccurate statement. a.One of the major difficulties visually impaired parents experience is the skepticism of health care professionals. b.Visually impaired mothers cannot overcome the infant's need for eye-to-eye contact. c.The best approach for the nurse is to assess the parents' capabilities rather than focusing on their disabilities. d.technologic advances, including the Internet, can provide deaf parents with a full range of parenting activities and information.

ANS: B Other sensory output can be provided by the parent, other people can participate, and other coping devices can be used. The skepticism, open or hidden, of health care professionals places an additional and unneeded hurdle for the parents. After the parents' capabilities have been assessed (including some the nurse may not have expected), the nurse can help find ways to assist the parents that play to their strengths. The Internet affords an extra teaching tool for the deaf, as do videos with subtitles or nurses signing. A number of electronic devices can turn sound into light flashes to help pick up a child's cry. Sign language is readily acquired by young children.

The nurse observes that a 15-year-old mother seems to ignore her newborn. A strategy that the nurse can use to facilitate mother-infant attachment in this mother is to: a.Tell the mother she must pay attention to her infant. b.Show the mother how the infant initiates interaction and pays attention to her. c.Demonstrate for the mother different positions for holding her infant while feeding. d.arrange for the mother to watch a video on parent-infant interaction.

ANS: B Pointing out the responsiveness of the infant is a positive strategy for facilitating parent-infant attachment. Telling the mother that she must pay attention to her infant may be perceived as derogatory and is not appropriate. Educating the young mother in infant care is important; however, pointing out the responsiveness of her baby is a better tool for facilitating mother-infant attachment. Videos are an educational tool that can demonstrate parent-infant attachment, but encouraging the mother to recognize the infant's responsiveness is more appropriate.

With regard to the adaptation of other family members, mainly siblings and grandparents, to the newborn, nurses should be aware that: a.Sibling rivalry cannot be dismissed as overblown psychobabble; negative feelings and behaviors can take a long time to blow over. b.Participation in preparation classes helps both siblings and grandparents. c.In the United States paternal and maternal grandparents consider themselves of equal importance and status. d.In the past few decades the number of grandparents providing permanent care to their grandchildren has been declining.

ANS: B Preparing older siblings and grandparents helps everyone to adapt. Sibling rivalry should be expected initially, but the negative behaviors associated with it have been overemphasized and stop in a comparatively short time. In the United States, in contrast to other cultures, paternal grandparents frequently consider themselves secondary to maternal grandparents. The number of grandparents providing permanent child care has been on the increase.

The postpartum woman who continually repeats the story of her labor, delivery, and recovery experience is: a.Providing others with her knowledge of events. b.Making the birth experience "real." c.Taking hold of the events leading to her labor and delivery. d.Accepting her response to labor and delivery.

ANS: B Reliving the birth experience makes the event real and helps the mother realize that the pregnancy is over and that the infant is born and is now a separate individual. The retelling of the story is to satisfy her needs, not the needs of others. This new mother is in the taking-in phase, trying to make the birth experience seem real and separate the infant from herself.

14. With regard to the postpartum changes and developments in a woman's cardiovascular system, nurses should be aware that: a. Cardiac output, the pulse rate, and stroke volume all return to prepregnancy normal values within a few hours of childbirth b. Respiratory function returns to nonpregnant levels by 6 to 8 weeks after birth c. The lowered white blood cell count after pregnancy can lead to false-positive results on tests for infections d. A hypercoagulable state protects the new mother from thromboembolism, especially after a cesarean birth

ANS: B Respirations should decrease to within the woman's normal prepregnancy range by 6 to 8 weeks after birth. Stroke volume increases and cardiac output remains high for a couple of days. However, the heart rate and blood pressure return to normal quickly. Leukocytosis increases 10 to 12 days after childbirth, which can obscure the diagnosis of acute infections (false-negative results). The hypercoagulable state increases the risk of thromboembolism, especially after a cesarean birth.

2. Which nursing intervention is paramount when providing care to a client with preterm labor who has received terbutaline? a. Assess deep tendon reflexes (DTRs). b. Assess for dyspnea and crackles. c. Assess for bradycardia. d. Assess for hypoglycemia.

ANS: B Terbutaline is a beta2-adrenergic agonist that affects the mother's cardiopulmonary and metabolic systems. Signs of cardiopulmonary decompensation include adventitious breath sounds and dyspnea. An assessment for dyspnea and crackles is important for the nurse to perform if the woman is taking magnesium sulfate. Assessing DTRs does not address the possible respiratory side effects of using terbutaline. Since terbutaline is a beta2-adrenergic agonist, it can lead to hyperglycemia, not hypoglycemia. Beta2-adrenergic agonist drugs cause tachycardia, not bradycardia. DIF: Cognitive Level: Analyze REF: pp. 767-768 TOP: Nursing Process: Assessment MSC: Client Needs: Physiologic Integrity

The nurse observes several interactions between a postpartum woman and her new son. What behavior, if exhibited by this woman, would the nurse identify as a possible maladaptive behavior regarding parent-infant attachment? a.talks and coos to her son b.Seldom makes eye contact with her son c.Cuddles her son close to her d.Tells visitors how well her son is feeding

ANS: B The woman should be encouraged to hold her infant in the en face position and make eye contact with the infant. Normal infant-parent interactions include talking and cooing to her son, cuddling her son close to her, and telling visitors how well her son is feeding.

8. A pregnant woman at 29 weeks of gestation has been diagnosed with preterm labor. Her labor is being controlled with tocolytic medications. She asks when she might be able to go home. Which response by the nurse is mostaccurate? a. "After the baby is born." b. "When we can stabilize your preterm labor and arrange home health visits." c. "Whenever your physician says that it is okay." d. "It depends on what kind of insurance coverage you have."

ANS: B This client's preterm labor is being controlled with tocolytics. Once she is stable, home care may be a viable option for this type of client. Care of a client with preterm labor is multidisciplinary and multifactorial; the goal is to prevent delivery. In many cases, this goal may be achieved at home. Managed care may dictate an earlier hospital discharge or a shift from hospital to home care. Insurance coverage may be one factor in client care, but ultimately, client safety remains the most important factor. DIF: Cognitive Level: Apply REF: p. 765 TOP: Nursing Process: Planning MSC: Client Needs: Health Promotion and Maintenance

10. What information should the nurse understand fully regarding rubella and Rh status? a. Breastfeeding mothers cannot be vaccinated with the live attenuated rubella virus. b. Women should be warned that the rubella vaccination is teratogenic and that they must avoid pregnancy for at least 1 month after vaccination. c. Rh immunoglobulin is safely administered intravenously because it cannot harm a nursing infant. d. Rh immunoglobulin boosts the immune system and thereby enhances the effectiveness of vaccinations.

ANS: B Women should understand that they must practice contraception for at least 1 month after being vaccinated. Because the live attenuated rubella virus is not communicable in breast milk, breastfeeding mothers can be vaccinated. Rh immunoglobulin is administered intramuscular (IM); it should never be administered to an infant. Rh immunoglobulin suppresses the immune system and therefore might thwart the rubella vaccination.

5. Indications for a primary cesarean birth are often nonrecurring. Therefore, a woman who has had a cesarean birth with a low transverse scar may be a candidate for vaginal birth after cesarean (VBAC). Which clients would beless likely to have a successful VBAC? (Select all that apply.) a. Lengthy interpregnancy interval b. African-American race c. Delivery at a rural hospital d. Estimated fetal weight <4000 g e. Maternal obesity (BMI >30)

ANS: B, C, E Indications for a low success rate for a VBAC delivery include a short interpregnancy interval, non-Caucasian race, gestational age longer than 40 weeks, maternal obesity, preeclampsia, fetal weight greater than 4000 g, and delivery at a rural or private hospital. DIF: Cognitive Level: Understand REF: p. 794 TOP: Nursing Process: Assessment MSC: Client Needs: Physiologic Integrity

5. Postpartum fatigue (PPF) is more than just feeling tired. It is a complex phenomenon affected by physiologic, psychologic, and situational variables. Which factors contribute to this phenomenon? (Select all that apply.) a. Precipitous labor b. Hospital routines c. Bottle feeding d. Anemia e. Excitement

ANS: B, D, E Physical fatigue and exhaustion are often associated with a long labor or cesarean birth, hospital routines, breastfeeding, and infant care. PPF is also attributed to anemia, infection, or thyroid dysfunction. The excitement and exhilaration of delivering a new infant along with well-intentioned visitors may make rest difficult.

MULTIPLE RESPONSE 1. The nurse recognizes that uterine hyperstimulation with oxytocin requires emergency interventions. What clinical cues alert the nurse that the woman is experiencing uterine hyperstimulation? (Select all that apply.) a. Uterine contractions lasting <90 seconds and occurring >2 minutes in frequency b. Uterine contractions lasting >90 seconds and occurring <2 minutes in frequency c. Uterine tone <20 mm Hg d. Uterine tone >20 mm Hg e. Increased uterine activity accompanied by a nonreassuring FHR and pattern

ANS: B, D, E Uterine contractions that occur less frequently than 2 minutes apart and last longer than 90 seconds, a uterine tone over 20 mm Hg, and a nonreassuring FHR and pattern are indications of uterine hyperstimulation with oxytocin administration. Uterine contractions that occur more frequently than 2 minutes apart and last less than 90 seconds are the expected goal of oxytocin induction. A uterine tone less than 20 mm Hg is normal. DIF: Cognitive Level: Analyze REF: p. 785 TOP: Nursing Process: Implementation MSC: Client Needs: Physiologic Integrity

Of the many factors that influence parental responses, nurses should be conscious of negative stereotypes that apply to specific patient populations. Which response could be an inappropriate stereotype of adolescent mothers? a.An adolescent mother's egocentricity and unmet developmental needs interfere with her ability to parent effectively. b.An adolescent mother is likely to use less verbal instruction, be less responsive, and interact less positively than other mothers. c.Adolescent mothers have a higher documented incidence of child abuse. d.Mothers older than 35 often deal with more stress related to work and career issues and decreasing libido.

ANS: C Adolescent mothers are more inclined to have a number of parenting difficulties that benefit from counseling; however, a higher incidence of child abuse is not one of them. Midlife mothers have many competencies, but they are more likely to have to deal with career issues and the accompanying stress.

21. What is the primary purpose for the use of tocolytic therapy to suppress uterine activity? a. Drugs can be efficaciously administered up to the designated beginning of term at 37 weeks gestation. b. Tocolytic therapy has no important maternal (as opposed to fetal) contraindications. c. The most important function of tocolytic therapy is to provide the opportunity to administer antenatal glucocorticoids. d. If the client develops pulmonary edema while receiving tocolytic therapy, then intravenous (IV) fluids should be given.

ANS: C Buying time for antenatal glucocorticoids to accelerate fetal lung development may be the best reason to use tocolytic therapy. Once the pregnancy has reached 34 weeks, however, the risks of tocolytic therapy outweigh the benefits. Important maternal contraindications to tocolytic therapy exist. Tocolytic-induced edema can be caused by IV fluids. DIF: Cognitive Level: Comprehend REF: p. 766 TOP: Nursing Process: Planning MSC: Client Needs: Physiologic Integrity

The nurse hears a primiparous woman talking to her son and telling him that his chin is just like his dad's chin. This woman's statement reflects: a.Mutuality. c.Claiming. b.Synchrony. d.Reciprocity.

ANS: C Claiming refers to the process by which the child is identified in terms of likeness to other family members. Mutuality occurs when the infant's behaviors and characteristics call forth a corresponding set of maternal behaviors and characteristics. Synchrony refers to the "fit" between the infant's cues and the parent's responses. Reciprocity is a type of body movement or behavior that provides the observer with cues.

The best way for the nurse to promote and support the maternal-infant bonding process is to: a.Help the mother identify her positive feelings toward the newborn. b.Encourage the mother to provide all newborn care. c.Assist the family with rooming-in. d.Return the newborn to the nursery during sleep periods.

ANS: C Close and frequent interaction between mother and infant, which is facilitated by rooming-in, is important in the bonding process. This is often referred to as the mother-baby care or couplet care. Having the mother express her feelings is important; however, it is not the best way to promote bonding. The mother needs time to rest and recuperate; she should not be expected to do all of the care. The patient needs to observe the infant during all stages so she will be aware of what to anticipate when they go home.

On observing a woman on her first postpartum day sitting in bed while her newborn lies awake in the bassinet, the nurse should: a.Realize that this situation is perfectly acceptable. b.Offer to hand the baby to the woman. c.Hand the baby to the woman. d.Explain "taking in" to the woman.

ANS: C During the "taking-in" phase of maternal adaptation (the mother may be passive and dependent), the nurse should encourage bonding when the infant is in the quiet alert stage. This is done best by simply giving the baby to the mother. The patient is exhibiting expected behavior during the taking-in phase; however, interventions by the nurse can facilitate infant bonding. The patient will learn best during the taking-hold phase.

During a phone follow-up conversation with a woman who is 4 days' postpartum, the woman tells the nurse, "I don't know what's wrong. I love my son, but I feel so let down. I seem to cry for no reason!" The nurse would recognize that the woman is experiencing: a.Taking-in. c,postpartum (PP) blues. b.Postpartum depression (PPD). d.Attachment difficulty.

ANS: C During the PP blues women are emotionally labile, often crying easily and for no apparent reason. This lability seems to peak around the fifth PP day. The taking-in phase is the period after birth when the mother focuses on her own psychologic needs. Typically this period lasts 24 hours. PPD is an intense, pervasive sadness marked by severe, labile mood swings; it is more serious and persistent than the PP blues. Crying is not a maladaptive attachment response; it indicates PP blues.

11. A recently delivered mother and her baby are at the clinic for a 6-week postpartum checkup. Which response by the client alerts the nurse that psychosocial outcomes have not been met? a. The woman excessively discusses her labor and birth experience. b. The woman feels that her baby is more attractive and clever than any others. c. The woman has not given the baby a name. d. The woman has a partner or family members who react very positively about the baby.

ANS: C If the mother is having difficulty naming her new infant, it may be a signal that she is not adapting well to parenthood. Other red flags include a refusal to hold or feed the baby, a lack of interaction with the infant, and becoming upset when the baby vomits or needs a diaper change. A new mother who is having difficulty is unwilling to discuss her labor and birth experience. An appropriate nursing diagnosis might be Impaired parenting, related to a long, difficult labor or unmet expectations of birth. A mother who is willing to discuss her birth experience is making a healthy personal adjustment. The mother who is not coping well finds her baby unattractive and messy. She may also be overly disappointed in the babys sex. The client might voice concern that the baby reminds her of a family member whom she does not like. Having a partner and/or other family members react positively is an indication that this new mother has a good support system in place. This support system helps reduce anxiety related to her new role as a mother.

16. Postpartum overdistention of the bladder and urinary retention can lead to which complications? a. Postpartum hemorrhage and eclampsia b. Fever and increased blood pressure c. Postpartum hemorrhage and urinary tract infection d. Urinary tract infection and uterine rupture

ANS: C Incomplete emptying and overdistention of the bladder can lead to urinary tract infection. Overdistention of the bladder displaces the uterus and prevents contraction of the uterine muscle, thus leading to postpartum hemorrhage. No correlation exists between bladder distention and high blood pressure or eclampsia. The risk of uterine rupture decreases after the birth of the infant.

With regard to parents' early and extended contact with their infant and the relationships built, nurses should be aware that: a.Immediate contact is essential for the parent-child relationship. b.Skin-to-skin contact is preferable to contact with the body totally wrapped in a blanket. c.Extended contact is especially important for adolescents and low-income women because they are at risk for parenting inadequacies. d.Mothers need to take precedence over their partners and other family matters.

ANS: C Nurses should encourage any activity that optimizes family extended contact. Immediate contact facilitates the attachment process but is not essential; otherwise, adopted infants would not establish the affectionate ties they do. The mode of infant-mother contact does not appear to have any important effect. Mothers and their partners are considered equally important.

The early postpartum period is a time of emotional and physical vulnerability. Many mothers can easily become psychologically overwhelmed by the reality of their new parental responsibilities. Fatigue compounds these issues. Although the baby blues are a common occurrence in the postpartum period, about one-half million women in America experience a more severe syndrome known as postpartum depression (PPD). Which statement regarding PPD is essential for the nurse to be aware of when attempting to formulate a nursing diagnosis? a.PPD symptoms are consistently severe. b.This syndrome affects only new mothers. c.PPD can easily go undetected. d.Only mental health professionals should teach new parents about this condition.

ANS: C PPD can go undetected because parents do not voluntarily admit to this type of emotional distress out of embarrassment, fear, or guilt. PPD symptoms range from mild to severe, with women having both good and bad days. Both mothers and fathers should be screened. PPD may also affect new fathers. The nurse should include information on PPD and how to differentiate this from the baby blues for all clients on discharge. Nurses also can urge new parents to report symptoms and seek follow-up care promptly if symptoms occur.

7. Prostaglandin gel has been ordered for a pregnant woman at 43 weeks of gestation. What is the primary purpose of prostaglandin administration? a. To enhance uteroplacental perfusion in an aging placenta b. To increase amniotic fluid volume c. To ripen the cervix in preparation for labor induction d. To stimulate the amniotic membranes to rupture

ANS: C Preparations of prostaglandin E1 and E2 are effective when used before labor induction to ripen (i.e., soften and thin) the cervix. Uteroplacental perfusion is not altered by the use of prostaglandins. The insertion of prostaglandin gel has no effect on the level of amniotic fluid. In some cases, women will spontaneously begin laboring after the administration of prostaglandins, thereby eliminating the need for oxytocin. It is not common for a woman's membranes to rupture as a result of prostaglandin use. DIF: Cognitive Level: Apply REF: p. 779 TOP: Nursing Process: Planning MSC: Client Needs: Physiologic Integrity

5. Which hormone remains elevated in the immediate postpartum period of the breastfeeding woman? a. Estrogen b. Progesterone c. Prolactin d. Human placental lactogen

ANS: C Prolactin levels in the blood increase progressively throughout pregnancy. In women who breastfeed, prolactin levels remain elevated into the sixth week after birth. Estrogen levels decrease markedly after expulsion of the placenta, reaching their lowest levels 1 week into the postpartum period. Progesterone levels decrease markedly after expulsion of the placenta, reaching their lowest levels 1 week into the postpartum period. Human placental lactogen levels dramatically decrease after expulsion of the placenta.

1. In planning for home care of a woman with preterm labor, which concern should the nurse need to address? a. Nursing assessments are different from those performed in the hospital setting. b. Restricted activity and medications are necessary to prevent a recurrence of preterm labor. c. Prolonged bed rest may cause negative physiologic effects. d. Home health care providers are necessary.

ANS: C Prolonged bed rest may cause adverse effects such as weight loss, loss of appetite, muscle wasting, weakness, bone demineralization, decreased cardiac output, risk for thrombophlebitis, alteration in bowel functions, sleep disturbance, and prolonged postpartum recovery. Nursing assessments differ somewhat from those performed in the acute care setting, but this concern does not need to be addressed. Restricted activity and medications may prevent preterm labor but not in all women. In addition, the plan of care is individualized to meet the needs of each client. Many women receive home health nurse visits, but care is individualized for each woman. DIF: Cognitive Level: Analyze REF: p. 777 TOP: Nursing Process: Planning MSC: Client Needs: Health Promotion and Maintenance

17. The nurse is teaching a client with preterm premature rupture of membranes (PPROM) regarding self-care activities. Which activities should the nurse include in her teaching? a. Report a temperature higher than 40° C. b. Tampons are safe to use to absorb the leaking amniotic fluid. c. Do not engage in sexual activity. d. Taking frequent tub baths is safe.

ANS: C Sexual activity should be avoided because it may induce preterm labor. A temperature higher than 38° C should be reported. To prevent the risk of infection, tub baths should be avoided and nothing should be inserted into the vagina. Further, foul-smelling vaginal fluid, which may be a sign of infection, should be reported. DIF: Cognitive Level: Apply REF: p. 762 TOP: Nursing Process: Implementation MSC: Client Needs: Health Promotion and Maintenance

5. A primigravida at 40 weeks of gestation is having uterine contractions every to 2 minutes and states that they are very painful. Her cervix is dilated 2 cm and has not changed in 3 hours. The woman is crying and wants an epidural. What is the likely status of this woman's labor? a. She is exhibiting hypotonic uterine dysfunction. b. She is experiencing a normal latent stage. c. She is exhibiting hypertonic uterine dysfunction. d. She is experiencing precipitous labor.

ANS: C The contraction pattern observed in this woman signifies hypertonic uterine activity. Typically, uterine activity in this phase occurs at 4- to 5-minute intervals lasting 30 to 45 seconds. Women who experience hypertonic uterine dysfunction, or primary dysfunctional labor, are often anxious first-time mothers who are having painful and frequent contractions that are ineffective at causing cervical dilation or effacement to progress. With hypotonic uterine dysfunction, the woman initially makes normal progress into the active stage of labor; then the contractions become weak and inefficient or stop altogether. Precipitous labor is one that lasts less than 3 hours from the onset of contractions until time of birth. DIF: Cognitive Level: Apply REF: p. 773 TOP: Nursing Process: Diagnosis MSC: Client Needs: Health Promotion and Maintenance

7. Under the Newborns and Mothers Health Protection Act, all health plans are required to allow new mothers and newborns to remain in the hospital for a minimum of _____ hours after a normal vaginal birth and for _____ hours after a cesarean birth. What is the correct interpretation of this legislation? a. 24; 72 b. 24; 96 c. 48; 96 d. 48; 120

ANS: C The specified stays are 48 hours (2 days) for a vaginal birth and 96 hours (4 days) for a cesarean birth. The attending provider and the mother together can decide on an earlier discharge. A client may be discharged either 24 hours after a vaginal birth or 72 hours after a cesarean birth if she is stable and her provider is in agreement. A client is unlikely to remain in the hospital for 120 hours after a cesarean birth unless complications have developed.

12. Which statement related to cephalopelvic disproportion (CPD) is the least accurate? a. CPD can be related to either fetal size or fetal position. b. The fetus cannot be born vaginally. c. CPD can be accurately predicted. d. Causes of CPD may have maternal or fetal origins.

ANS: C Unfortunately, accurately predicting CPD is not possible. Although CPD is often related to excessive fetal size (macrosomia), malposition of the fetal presenting part is the problem in many cases, not true CPD. When CPD is present, the fetus cannot fit through the maternal pelvis to be born vaginally. CPD may be related to either fetal origins such as macrosomia or malposition or maternal origins such as a too small or malformed pelvis. DIF: Cognitive Level: Understand REF: p. 775 TOP: Nursing Process: Planning MSC: Client Needs: Health Promotion and Maintenance

6. A woman is having her first child. She has been in labor for 15 hours. A vaginal examination performed 2 hours earlier revealed the cervix to be dilated to 5 cm and 100% effaced, and the presenting part of the fetus was at station 0; however, another vaginal examination performed 5 minutes ago indicated no changes. What abnormal labor pattern is associated with this description? a. Prolonged latent phase b. Protracted active phase c. Secondary arrest d. Protracted descent

ANS: C With a secondary arrest of the active phase, the progress of labor has stopped. This client has not had any anticipated cervical change, indicating an arrest of labor. In the nulliparous woman, a prolonged latent phase typically lasts longer than 20 hours. A protracted active phase, the first or second stage of labor, is prolonged (slow dilation). With a protracted descent, the fetus fails to descend at an anticipated rate during the deceleration phase and second stage of labor. DIF: Cognitive Level: Analyze REF: p. 774 TOP: Nursing Process: Assessment MSC: Client Needs: Health Promotion and Maintenance

6. Two days ago a woman gave birth to a full-term infant. Last night she awakened several times to urinate and noted that her gown and bedding were wet from profuse diaphoresis. One mechanism for the diaphoresis and diuresis that this woman is experiencing during the early postpartum period is: a. Elevated temperature caused by postpartum infection b. Increased basal metabolic rate after giving birth c. Loss of increased blood volume associated with pregnancy d. Increased venous pressure in the lower extremities

ANS: C Within 12 hours of birth, women begin to lose the excess tissue fluid that has accumulated during pregnancy. One mechanism for reducing these retained fluids is the profuse diaphoresis that often occurs, especially at night, for the first 2 or 3 days after childbirth. Postpartal diuresis is another mechanism by which the body rids itself of excess fluid. An elevated temperature causes chills and possibly dehydration, not diaphoresis and diuresis. Diaphoresis and diuresis sometimes are referred to as reversal of the water metabolism of pregnancy, not as the basal metabolic rate. Postpartal diuresis may be caused by the removal of increased venous pressure in the lower extremities.

A 25-year-old gravida 1 para 1 who had an emergency cesarean birth 3 days ago is scheduled for discharge. As you prepare her for discharge, she begins to cry. Your initial action should be to a.Assess her for pain. b.Point out how lucky she is to have a healthy baby. c.Explain that she is experiencing postpartum blues. d.allow her time to express her feelings.

ANS: D Although many women experience transient postpartum blues, they need assistance in expressing their feelings. This condition affects 50% to 80% of new mothers. There should be no assumption that the patient is in pain, when in fact she may have no pain whatsoever. This is "blocking" communication and inappropriate in this situation. The patient needs the opportunity to express her feelings first; patient teaching can occur later.

7. A nurse caring for a postpartum woman understands that breast engorgement is caused by: a. Overproduction of colostrum b. Accumulation of milk in the lactiferous ducts and glands c. Hyperplasia of mammary tissue d. Congestion of veins and lymphatics

ANS: D Breast engorgement is caused by the temporary congestion of veins and lymphatics. Test Bank Mosby items and derived items © 2012, 2007, 2006 by Mosby, Inc., an affiliate of Elsevier Inc. 20-4 Breast engorgement is not the result of overproduction of colostrum. Accumulation of milk in the lactiferous ducts and glands does not cause breast engorgement. Hyperplasia of mammary tissue does not cause breast engorgement.

12. Parents who have not already done so need to make time for newborn follow-up of the discharge. According to the American Academy of Pediatrics (AAP), when should a breastfeeding infant first need to be seen for a follow-up examination? a. 2 weeks of age b. 7 to 10 days after childbirth c. 4 to 5 days after hospital discharge d. 48 to 72 hours after hospital discharge

ANS: D Breastfeeding infants are routinely seen by the pediatric health care provider clinic within 3 to 5 days after birth or 48 to 72 hours after hospital discharge and again at 2 weeks of age. Formula-feeding infants may be seen for the first time at 2 weeks of age.

Nursing activities that promote parent-infant attachment are many and varied. One activity that should not be overlooked is management of the environment. While providing routine mother-baby care, the nurse should ensure that: a.The baby is able to return to the nursery at night so that the new mother can sleep. b.Routine times for care are established to reassure the parents. c.The father should be encouraged to go home at night to prepare for mother-baby discharge. d.An environment that fosters as much privacy as possible should be created.

ANS: D Care providers need to knock before gaining entry. Nursing care activities should be grouped. Once the baby has demonstrated adjustment to extrauterine life (either in the mother's room or the transitional nursery), all care should be provided in one location. This important principle of family-centered maternity care fosters attachment by giving parents the opportunity to learn about their infant 24 hours a day. One nurse should provide care to both mother and baby in this couplet care or rooming-in model. It is not necessary for the baby to return to the nursery at night. In fact, the mother will sleep better with the infant close by. Care should be individualized to meet the parents' needs, not the routines of the staff. Teaching goals should be developed in collaboration with the parents. The father or other significant other should be permitted to sleep in the room with the mother. The maternity unit should develop policies that allow for the presence of significant others as much as the new mother desires.

18. A woman at 26 weeks of gestation is being assessed to determine whether she is experiencing preterm labor. Which finding indicates that preterm labor is occurring? a. Estriol is not found in maternal saliva. b. Irregular, mild uterine contractions are occurring every 12 to 15 minutes. c. Fetal fibronectin is present in vaginal secretions. d. The cervix is effacing and dilated to 2 cm.

ANS: D Cervical changes such as shortened endocervical length, effacement, and dilation are predictors of imminent preterm labor. Changes in the cervix accompanied by regular contractions indicate labor at any gestation. Estriol is a form of estrogen produced by the fetus that is present in plasma at 9 weeks of gestation. Levels of salivary estriol have been shown to increase before preterm birth. Irregular, mild contractions that do not cause cervical change are not considered a threat. The presence of fetal fibronectin in vaginal secretions between 24 and 36 weeks of gestation could predict preterm labor, but it has only a 20% to 40% positive predictive value. Of more importance are other physiologic clues of preterm labor such as cervical changes. DIF: Cognitive Level: Apply REF: p. 759 TOP: Nursing Process: Assessment | Nursing Process: Planning MSC: Client Needs: Health Promotion and Maintenance

11. The nurse is performing an assessment on a client who thinks she may be experiencing preterm labor. Which information is the most important for the nurse to understand and share with the client? a. Because all women must be considered at risk for preterm labor and prediction is so variable, teaching pregnant women the symptoms of preterm labor probably causes more harm through false alarms. b. Braxton Hicks contractions often signal the onset of preterm labor. c. Because preterm labor is likely to be the start of an extended labor, a woman with symptoms can wait several hours before contacting the primary caregiver. d. Diagnosis of preterm labor is based on gestational age, uterine activity, and progressive cervical change.

ANS: D Gestational age of 20 to 37 weeks, uterine contractions, and a cervix that is 80% effaced or dilated 2 cm indicates preterm labor. It is essential that nurses teach women how to detect the early symptoms of preterm labor. Braxton Hicks contractions resemble preterm labor contractions, but they are not true labor. Waiting too long to see a health care provider could result in essential medications failing to be administered. Preterm labor is not necessarily long-term labor. DIF: Cognitive Level: Understand REF: p. 759 TOP: Nursing Process: Planning MSC: Client Needs: Safe and Effective Care Environment

After giving birth to a healthy infant boy, a primiparous woman, 16 years old, is admitted to the postpartum unit. An appropriate nursing diagnosis for her at this time is Risk for impaired parenting related to deficient knowledge of newborn care. In planning for the woman's discharge, what should the nurse be certain to include in the plan of care? a.Instruct the patient how to feed and bathe her infant. b.Give the patient written information on bathing her infant. c.Advise the patient that all mothers instinctively know how to care for their infants. d.Provide time for the patient to bathe her infant after she views an infant bath demonstration.

ANS: D Having the mother demonstrate infant care is a valuable method of assessing the client's understanding of her newly acquired knowledge, especially in this age group, because she may inadvertently neglect her child. Although verbalizing how to care for the infant is a form of patient education, it is not the most developmentally appropriate teaching for a teenage mother. Advising the patient that all mothers instinctively know how to care for their infants is an inappropriate statement; it is belittling and false.

15. Which condition, not uncommon in pregnancy, is likely to require careful medical assessment during the puerperium? a. Varicosities of the legs b. Carpal tunnel syndrome c. Periodic numbness and tingling of the fingers d. Headaches

ANS: D Headaches in the postpartum period can have a number of causes, some of which deserve medical attention. Total or nearly total regression of varicosities is expected after childbirth. However, headaches might deserve attention. Carpal tunnel syndrome is relieved in childbirth when the compression on the median nerve is lessened. Headaches, however, might deserve attention. Periodic numbness of the fingers usually disappears after birth unless carrying the baby aggravates the condition. Headaches, however, might deserve attention.

13. Which statement related to the induction of labor is most accurate? a. Can be achieved by external and internal version techniques b. Is also known as a trial of labor (TOL) c. Is almost always performed for medical reasons d. Is rated for viability by a Bishop score

ANS: D Induction of labor is likely to be more successful with a Bishop score of 9 or higher for first-time mothers or 5 or higher for veterans. Version is the turning of the fetus to a better position by a physician for an easier or safer birth. A TOL is the observance of a woman and her fetus for several hours of active labor to assess the safety of vaginal birth. Two thirds of cases of induced labor are elective and not done for medical reasons. DIF: Cognitive Level: Understand REF: p. 780 TOP: Nursing Process: Diagnosis MSC: Client Needs: Safe and Effective Care Environment

22. When would an internal version be indicated to manipulate the fetus into a vertex position? a. Fetus from a breech to a cephalic presentation before labor begins b. Fetus from a transverse lie to a longitudinal lie before a cesarean birth c. Second twin from an oblique lie to a transverse lie before labor begins d. Second twin from a transverse lie to a breech presentation during a vaginal birth

ANS: D Internal version is used only during a vaginal birth to manipulate the second twin into a presentation that allows it to be vaginally born. For internal version to occur, the cervix needs to be completely dilated. DIF: Cognitive Level: Remember REF: p. 779 TOP: Nursing Process: Assessment MSC: Client Needs: Physiologic Integrity

9. Because a full bladder prevents the uterus from contracting normally, nurses intervene to help the woman spontaneously empty her bladder as soon as possible. If all else fails, what tactic might the nurse use? a. Pouring water from a squeeze bottle over the womans perineum b. Placing oil of peppermint in a bedpan under the woman c. Asking the physician to prescribe analgesic agents d. Inserting a sterile catheter

ANS: D Invasive procedures are usually the last to be tried, especially with so many other simple and easy methods available (e.g., water, peppermint vapors, pain pills). Pouring water over the perineum may stimulate voiding. It is easy, noninvasive, and should be tried first. The oil of peppermint releases vapors that may relax the necessary muscles. It, too, is easy, noninvasive, and should be tried early on. If the woman is anticipating pain from voiding, then pain medications may be helpful. Other nonmedical means should be tried first, but medications still come before the insertion of a catheter.

4. A woman gave birth to a healthy infant boy 5 days ago. What type of lochia does the nurse expect to find when assessing this woman? a. Lochia rubra b. Lochia sangra c. Lochia alba d. Lochia serosa

ANS: D Lochia serosa, which consists of blood, serum, leukocytes, and tissue debris, generally occurs around day 3 or 4 after childbirth. Lochia rubra consists of blood and decidual and trophoblastic debris. The flow generally lasts 3 to 4 days and pales, becoming pink or brown. Lochia sangra is not a real term. Lochia alba occurs in most women after day 10 and can continue up to 6 weeks after childbirth.

6. The trend in the United States is for women to remain hospitalized no longer than 1 or 2 days after giving birth. Which scenario is not a contributor to this model of care? a. Wellness orientation model of care rather than a sick-care model b. Desire to reduce health care costs c. Consumer demand for fewer medical interventions and more family-focused experiences d. Less need for nursing time as a result of more medical and technologic advances and devices available at home that can provide information

ANS: D Nursing time and care are in demand as much as ever; the nurse simply has to do things more quickly. A wellness orientation model of care seems to focus on getting clients out the door sooner. In most cases, less hospitalization results in lower costs. People believe that the family gives more nurturing care than the institution.

4. A 25-year-old multiparous woman gave birth to an infant boy 1 day ago. Today her husband brings a large container of brown seaweed soup to the hospital. When the nurse enters the room, the husband asks for help with warming the soup so that his wife can eat it. What is the nurses most appropriate response? a. Didnt you like your lunch? b. Does your physician know that you are planning to eat that? c. What is that anyway? d. I'll warm the soup in the microwave for you.

ANS: D Offering to warm the food shows cultural sensitivity to the dietary preferences of the woman and is the most appropriate response. Cultural dietary preferences must be respected. Women may request that family members bring favorite or culturally appropriate foods to the hospital. Asking the woman to identify her food does not show cultural sensitivity.

19. Which assessment is least likely to be associated with a breech presentation? a. Meconium-stained amniotic fluid b. Fetal heart tones heard at or above the maternal umbilicus c. Preterm labor and birth d. Postterm gestation

ANS: D Postterm gestation is not likely to occur with a breech presentation. The presence of meconium in a breech presentation may be a result of pressure on the fetal wall as it traverses the birth canal. Fetal heart tones heard at the level of the umbilical level of the mother are a typical finding in a breech presentation because the fetal back would be located in the upper abdominal area. Breech presentations often occur in preterm births. DIF: Cognitive Level: Analyze REF: pp. 775-776 TOP: Nursing Process: Assessment MSC: Client Needs: Health Promotion and Maintenance

2. To provide optimum care for the postpartum woman, the nurse understands that the most common causes of subinvolution are: a. Postpartum hemorrhage and infection b. Multiple gestation and postpartum hemorrhage c. Uterine tetany and overproduction of oxytocin d. Retained placental fragments and infection

ANS: D Subinvolution is failure of the uterus to return to a nonpregnant state. The most common causes of subinvolution are retained placental fragments and infection. Subinvolution may be caused by infection and result in hemorrhage. Multiple gestation may cause uterine atony, resulting in postpartum hemorrhage. Uterine tetany and overproduction of oxytocin do not cause subinvolution.

9. The obstetric provider has informed the nurse that she will be performing an amniotomy on the client to induce labor. What is the nurse's highest priority intervention after the amniotomy is performed? a. Applying clean linens under the woman b. Taking the client's vital signs c. Performing a vaginal examination d. Assessing the fetal heart rate (FHR)

ANS: D The FHR is assessed before and immediately after the amniotomy to detect any changes that might indicate cord compression or prolapse. Providing comfort measures, such as clean linens, for the client is important but not the priority immediately after an amniotomy. The woman's temperature should be checked every 2 hours after the rupture of membranes but not the priority immediately after an amniotomy. The woman would have had a vaginal examination during the procedure. Unless cord prolapse is suspected, another vaginal examination is not warranted. Additionally, FHR assessment provides clinical cues to a prolapsed cord. DIF: Cognitive Level: Analyze REF: p. 783 TOP: Nursing Process: Implementation MSC: Client Needs: Physiologic Integrity

12. With regard to postpartum ovarian function, nurses should be aware that: a. Almost 75% of women who do not breastfeed resume menstruating within a month after birth b. Ovulation occurs slightly earlier for breastfeeding women c. Because of menstruation/ovulation schedules, contraception considerations can be postponed until after the puerperium d. The first menstrual flow after childbirth usually is heavier than normal

ANS: D The first flow is heavier, but within three or four cycles it is back to normal. Ovulation can occur within the first month, but for 70% of nonlactating women, it returns in about 3 months. Breastfeeding women take longer to resume ovulation. Because many women ovulate before their first postpartum menstrual period, contraceptive options need to be discussed early in the puerperium.

After they are born, a crying infant may be soothed by being held in a position in which the newborn can hear the mother's heartbeat. This phenomenon is known as: a.Entrainment. c.Synchrony. b.Reciprocity. d.Biorhythmicity.

ANS: D The newborn is in rhythm with the mother. The infant develops a personal biorhythm with the parents' help over time. Entrainment is the movement of newborns in time to the structure of adult speech. Reciprocity is body movement or behavior that gives cues to the person's desires. These take several weeks to develop with a new baby. Synchrony is the fit between the infant's behavioral cues and the parent's responses.

A new father states, "I know nothing about babies," but he seems to be interested in learning. This is an ideal opportunity for the nurse to: a.Continue to observe his interaction with the newborn. b.Tell him when he does something wrong. c.Show no concern, as he will learn on his own. d.Include him in teaching sessions.

ANS: D The nurse must be sensitive to the father's needs and include him whenever possible. As fathers take on their new role, the nurse should praise every attempt, even if his early care is awkward. It is important to note the bonding process of the mother and the father; however, that does not satisfy the expressed needs of the father. The new father should be encouraged in caring for his baby by pointing out the things that he does right. Criticizing him will discourage him.

13. On examining a woman who gave birth 5 hours ago, the nurse finds that the woman has completely saturated a perineal pad within 15 minutes. What is the nurses highest priority at this time? a. Beginning an intravenous (IV) infusion of Ringers lactate solution b. Assessing the womans vital signs c. Calling the womans primary health care provider d. Massaging the womans fundus

ANS: D The nurse should first assess the uterus for atony by massaging the womans fundus. Uterine tone must be established to prevent excessive blood loss. The nurse may begin an IV infusion to restore circulatory volume, but this would not be the first action. Blood pressure is not a reliable indicator of impending shock from impending hemorrhage; assessing vital signs should not be the nurses first action. The physician would be notified after the nurse completes the assessment of the woman.

A nurse is observing a family. The mother is holding the baby she delivered less than 24 hours ago. Her husband is watching his wife and asking questions about newborn care. The 4-year-old brother is punching his mother on the back. The nurse should: a.Report the incident to the social services department. b.Advise the parents that the toddler needs to be reprimanded. c.Report to oncoming staff that the mother is probably not a good disciplinarian. d.Realize that this is a normal family adjusting to family change.

ANS: D The observed behaviors are normal variations of families adjusting to change. There is no need to report this one incident. Giving advice at this point would make the parents feel inadequate.

9. The interval between the birth of the newborn and the return of the reproductive organs to their normal nonpregnant state is called the: a. Involutionary period because of what happens to the uterus b. Lochia period because of the nature of the vaginal discharge c. Mini-tri period because it lasts only 3 to 6 weeks d. Puerperium, or fourth trimester of pregnancy

ANS: D The puerperium, also called the fourth trimester or the postpartum period of pregnancy, lasts about 3 to 6 weeks. Involution marks the end of the puerperium, or the fourth trimester of pregnancy. Lochia refers to the various vaginal discharges during the puerperium, or fourth trimester of pregnancy. The final period of pregnancy lasts 3 to 6 weeks, but it is called the puerperium, or fourth trimester of pregnancy.

3. In evaluating the effectiveness of magnesium sulfate for the treatment of preterm labor, which finding alerts the nurse to possible side effects? a. Urine output of 160 ml in 4 hours b. DTRs 2+ and no clonus c. Respiratory rate (RR) of 16 breaths per minute d. Serum magnesium level of 10 mg/dl

ANS: D The therapeutic range for magnesium sulfate management is 4 to 7.5 mg/dl. A serum magnesium level of 10 mg/dl could lead to signs and symptoms of magnesium toxicity, including oliguria and respiratory distress. Urine output of 160 ml in 4 hours, DTRs of 2+, and a RR of 16 breaths per minute are all normal findings. DIF: Cognitive Level: Apply REF: p. 767 TOP: Nursing Process: Evaluation MSC: Client Needs: Physiologic Integrity

17. Childbirth may result in injuries to the vagina and uterus. Pelvic floor exercises, also known as Kegel exercises, will help to strengthen the perineal muscles and encourage healing. The nurse knows that the client understands the correct process for completing these conditioning exercises when she reports: a. "I contract my thighs, buttocks, and abdomen." b. "I do 10 of these exercises every day." c. "I stand while practicing this new exercise routine." d. "I pretend that I am trying to stop the flow of urine midstream."

ANS: D The woman can pretend that she is attempting to stop the passing of gas or the flow of urine midstream. This will replicate the sensation of the muscles drawing upward and inward. Each contraction should be as intense as possible without contracting the abdomen, buttocks, or thighs. Guidelines suggest that these exercises should be done 24 to 100 times per day. Positive results are shown with a minimum of 24 to 45 repetitions per day. The best position to learn Kegel exercises is to lie supine with knees bent. A secondary position is on the hands and knees.

1. A woman gave birth vaginally to a 9-pound, 12-ounce girl yesterday. Her primary health care provider has written orders for perineal ice packs, use of a sitz bath three times daily, and a stool softener. Which information regarding the clients condition is most closely correlated with these orders? a. Woman is a gravida 2, para 2. b.Woman had a vacuum-assisted birth. c.Woman received epidural anesthesia. d.Woman has an episiotomy.

ANS: D These orders are typical interventions for a woman who has had an episiotomy, lacerations, and hemorrhoids. A multiparous classification is not an indication for these orders. A vacuum-assisted birth may be used in conjunction with an episiotomy, which would indicate these interventions. The use of an epidural anesthesia has no correlation with these orders.

18. Which nursing action is most appropriate to correct a boggy uterus that is displaced above and to the right of the umbilicus? a. Notify the physician of an impending hemorrhage. b. Assess the blood pressure and pulse. c. Evaluate the lochia. d. Assist the client in emptying her bladder.

ANS: D Urinary retention may cause overdistention of the urinary bladder, which lifts and displaces the uterus. Nursing actions need to be implemented before notifying the physician. Evaluating blood pressure, pulse, and lochia is important if the bleeding continues; however, the focus at this point is to assist the client in emptying her bladder.

____________________ is the process by which the parent and infant come to love and accept each other.

Attachment p. 612

The nurse observes that a 15-year-old mother seems to ignore her newborn. A strategy that the nurse can use to facilitate mother-infant attachment in this mother is: A. Tell the mother she must pay attention to her infant B. Show the mother how the infant initiates interaction and attends to her C. Demonstrate for the mother different positions for holding her infant while feeding D. Arrange for the mother to watch a video on parent-infant interaction

B A. Incorrect: A statement of this kind may be perceived as derogatory and is not appropriate. B. Correct: Pointing out the responsiveness of the infant is a positive strategy for facilitating parent-infant attachment. C. Incorrect: Educating the young mother in infant care is important, but pointing out the responsiveness of her baby is a better tool for facilitating mother-infant attachment. D. Incorrect: Videos are an educational tool that can demonstrate parent-infant attachment, but encouraging the mother to recognize the infant's responsiveness is more appropriate. p. 624

Other early sensual contacts between infant and mother involve sound and smell. Nurses should be aware that despite what folk wisdom might say: A. High-pitched voices irritate newborns. B. Infants can learn to distinguish their mother's voice from others soon after birth. C. All babies in the hospital smell alike. D. A mother's breast milk has no distinctive odor.

B A. Incorrect: Infants respond positively to high-pitched voices. B. Correct: Infants know the sound of their mother's voice early. C. Incorrect: Each infant has a unique odor. D. Incorrect: Infants quickly learn to distinguish the odor of their mother's breast milk. p. 617

Many first-time parents do not plan on their parents' help immediately after the newborn arrives. What statement by the nurse is the most appropriate when counseling new parents about the involvement of grandparents? A. "You should tell your parents to leave you alone." B. "Grandparents can help you with parenting skills and also help preserve family traditions." C. "Grandparent involvement can be very disruptive to the family." D. "They are getting old. You should let them be involved while they can."

B A. Incorrect: Intergenerational help may be perceived as interference, but a statement of this sort is not therapeutic to the adaptation of the family. B. Correct: This is the most appropriate response. C. Incorrect: Not only is this statement invalid, it also is not an appropriate nursing response. D. Incorrect: Regardless of age, grandparents can help with parenting skills and preserve family traditions. This is not the most appropriate statement, and it does not demonstrate sensitivity on the part of the nurse. p. 631

With regard to the adaptation of other family members, mainly siblings and grandparents, to the newborn, nurses should be aware that: A. Sibling rivalry cannot be dismissed as overblown psychobabble; negative feelings and behaviors can take a long time to blow over. B. Participation in preparation classes helps both siblings and grandparents. C. In the United States, paternal and maternal grandparents consider themselves of equal importance and status. D. Since 1990, the number of grandparents providing permanent care to their grandchildren has been declining.

B A. Incorrect: Sibling rivalry should be expected initially, but the negative behaviors associated with it have been overemphasized and stop in a comparatively short time. B. Correct: Preparing older siblings, as well as grandparents, helps with everyone's adaptation. C. Incorrect: In the United States, in contrast to other cultures, paternal grandparents frequently consider themselves secondary to maternal grandparents. D. Incorrect: The number of grandparents providing permanent child care has been rising. pp. 629-630

When dealing with parents who have some form of sensory impairment, nurses should realize that all of these statements are true except: A. One of the major difficulties visually impaired parents experience is the skepticism of health care professionals. B. Visually impaired mothers cannot overcome the infant's need for eye-to-eye contact. C. The best approach for the nurse is to assess the parents' capabilities rather than focusing on their disabilities. D. Technologic advances, including the Internet, can provide deaf parents with a full range of parenting activities and information.

B A. Incorrect: The skepticism, open or hidden, of health care professionals throws up an additional and unneeded hurdle for the parents. B. Correct: Other sensory output can be provided by the parent, other people can participate, and other coping devices can be used. C. Incorrect: After the parents' capabilities have been assessed (including some the nurse may not have expected), the nurse can help find ways to assist the parents that play to their strengths. D. Incorrect: The Internet affords an extra teaching tool for the deaf, as do videos with subtitles or nurses signing. A number of electronic devices can turn sound into light flashes to help pick up a child's cry. Sign language is acquired readily by young children. p. 628

The nurse observes several interactions between a postpartum woman and her new son. What behavior, if exhibited by this woman, would the nurse identify as a possible maladaptive behavior regarding parent-infant attachment? A. Talks and coos to her son B. Seldom makes eye contact with her son C. Cuddles her son close to her D. Tells visitors how well her son is feeding

B A. Incorrect: This is a normal infant-parent interaction. B. Correct: The woman should be encouraged to hold her infant in the en face position and make eye contact with the infant. C. Incorrect: This is a normal infant-parent interaction. D. Incorrect: This is a normal infant-parent interaction. p. 625

In follow-up appointments or visits with parents and their new baby, it may be useful if the nurse can identify parental behaviors that can either facilitate or inhibit attachment. What is a facilitating behavior? A. The parents have difficulty naming the infant. B. The parents hover around the infant, directing attention to and pointing at the infant. C. The parents make no effort to interpret the actions or needs of the infant. D. The parents do not move from fingertip touch to palmar contact and holding.

B A. Incorrect: This would be an inhibiting behavior. B. Correct: Hovering over the infant, as well as obviously paying attention to the baby, is a facilitating behavior. C. Incorrect: This would be an inhibiting behavior. D. Incorrect: This would be an inhibiting behavior. p. 614

Which client is at greatest risk for early PPH? a. Primiparous woman (G 2, P 1-0-0-1) being prepared for an emergency cesarean birth for fetal distress b. Woman with severe preeclampsia on magnesium sulfate whose labor is being induced c. Multiparous woman (G 3, P 2-0-0-2) with an 8-hour labor d. Primigravida in spontaneous labor with preterm twins

B Magnesium sulfate administration during labor poses a risk for PPH. Magnesium acts as a smooth muscle relaxant, thereby contributing to uterine relaxation and atony. A primiparous woman being prepared for an emergency cesarean birth for fetal distress, a multiparous woman with an 8-hour labor, and a primigravida in spontaneous labor with preterm twins do not indicate risk factors or causes of early PPH.

What is the primary nursing responsibility when caring for a client who is experiencing an obstetric hemorrhage associated with uterine atony? a. Establishing venous access b. Performing fundal massage c. Preparing the woman for surgical intervention d. Catheterizing the bladder

B The initial management of excessive postpartum bleeding is a firm massage of the uterine fundus. Although establishing venous access may be a necessary intervention, fundal massage is the initial intervention. The woman may need surgical intervention to treat her postpartum hemorrhage, but the initial nursing intervention is to assess the uterus. After uterine massage, the nurse may want to catheterize the client to eliminate any bladder distention that may be preventing the uterus from properly contracting.

What would a steady trickle of bright red blood from the vagina in the presence of a firm fundus suggest to the nurse? a.Uterine atony b.Lacerations of the genital tract c. Perineal hematoma d. Infection of the uterus

B Undetected lacerations will bleed slowly and continuously. Bleeding from lacerations is uncontrolled by uterine contraction. The fundus is not firm in the presence of uterine atony. A hematoma would develop internally. Swelling and discoloration would be noticeable; however, bright bleeding would not be. With an infection of the uterus, an odor to the lochia and systemic symptoms such as fever and malaise would be present.

The breast-feeding mother should be taught to expect which changes to the condition of the breasts? (Select all that apply.) a. Breast tenderness is likely to persist for approximately 1 week after the start of lactation. b. As lactation is established, a mass may form that can be distinguished from cancer by its positional shift from day to day. c. In nonlactating mothers, colostrum is present for the first few days after childbirth. d. If suckling is never begun or is discontinued, then lactation ceases within a few days to a week. e. Little change occurs to the breasts in the first 48 hours.

B, C, D Breasts become fuller and heavier as colostrum transitions to milk; this fullness should last 72 to 96 hours. The movable, noncancerous mass is a filled milk sac. Colostrum is present for a few days whether or not the mother breastfeeds. A mother who does not want to breastfeed should also avoid stimulating her nipples. Little change to the breasts occurs in the first 24 hours of childbirth.

Of the many factors that influence parental responses, nurses should be aware that all of these statements regarding age are true except: A. An adolescent mother's egocentricity and unmet developmental needs interfere with her ability to parent effectively. B. An adolescent mother is likely to use less verbal instruction, be less responsive, and interact less positively than other mothers. C. Adolescent mothers have a higher documented incidence of child abuse. D. Mothers older than 35 often deal with more stress related to work and career issues, as well as decreasing libido.

C A. Incorrect: Adolescent mothers are more inclined to have a number of parenting difficulties that benefit from counseling, but a higher incidence of child abuse is not one of them. B. Incorrect: Adolescent mothers are more inclined to have a number of parenting difficulties that benefit from counseling, but a higher incidence of child abuse is not one of them. C. Correct: Adolescent mothers are more inclined to have a number of parenting difficulties that benefit from counseling, but a higher incidence of child abuse is not one of them. D. Incorrect: Midlife mothers have many competencies but are more likely to have to deal with career and sexual issues than are younger mothers. p. 625

With regard to parents' early and extended contact with their infant and the relationships built, nurses should be aware that: A. Immediate contact is essential for the parent-child relationship. B. Skin-to-skin contact is preferable to contact with the body totally wrapped in a blanket. C. Extended contact is especially important for adolescents and low-income women because they are at risk for parenting inadequacies. D. Mothers need to take precedence over their partners and other family matters.

C A. Incorrect: Immediate contact facilitates the attachment process but is not essential; otherwise, adopted infants would not establish the affectionate ties they do. B. Incorrect: The mode of infant-mother contact does not appear to have any important effect. C. Correct: Nurses should encourage any activity that optimizes family extended contact. D. Incorrect: Mothers and their partners are considered equally important. p. 616

The nurse hears a primiparous woman talking to her son and telling him that his chin is just like his dad's chin. This woman's statement reflects: A. Mutuality B. Synchrony C. Claiming D. Reciprocity

C A. Incorrect: Mutuality occurs when the infant's behaviors and characteristics call forth a corresponding set of maternal behaviors and characteristics. B. Incorrect: Synchrony refers to the "fit" between the infant's cues and the parent's responses. C. Correct: Claiming refers to the process by which the child is identified in terms of likeness to other family members. D. Incorrect: Reciprocity is a type of body movement or behavior that provides the observer with cues. p. 613

The early postpartum period is a time of emotional and physical vulnerability. Many mothers can easily become psychologically overwhelmed by the reality of their new parental responsibilities. Fatigue compounds these issues. Although the baby blues are a common occurrence in the postpartum period, about one-half million women in America experience a more severe syndrome known as postpartum depression. Which statement regarding postpartum depression (PPD) is essential for the nurse to be aware of when attempting to formulate a nursing diagnosis? A. PPD symptoms are consistently severe B. This syndrome affects only new mothers C. PPD can easily go undetected D. Only mental health professionals should teach new parents about this condition

C A. Incorrect: PPD symptoms range from mild to severe, with women having both good day and bad days. B. Incorrect: Screening should be done for both mothers and fathers. PPD in new fathers ranges from 1% to 26%. C. Correct: PPD can go undetected because parents do not voluntarily admit to this type of emotional distress out of embarrassment, fear, or guilt. D. Incorrect: The nurse should include information on PPD and how to differentiate this from the baby blues for all clients on discharge. Nurses also can urge new parents to report symptoms and seek follow-up care promptly if they occur. pp. 621, 622

During a phone follow-up conversation with a woman who is 4 days postpartum, the woman tells the nurse, "I don't know what's wrong. I love my son, but I feel so let down. I seem to cry for no reason!" The nurse would recognize that the woman is experiencing: A. Taking-in B. Postpartum depression (PPD) C. Postpartum blues D. Attachment difficulty

C A. Incorrect: The taking-in phase is the period after birth when the mother focuses on her own psychologic needs. Typically, this period lasts 24 hours. B. Incorrect: PPD is an intense, pervasive sadness marked by severe, labile mood swings; it is more serious and persistent than the PP blues. C. Correct: During the PP blues, women are emotionally labile, often crying easily and for no apparent reason. This lability seems to peak around the fifth PP day. D. Incorrect: Crying is not a maladaptive attachment response; it indicates PP blues. p. 620

Which is the initial treatment for the client with vWD who experiences a PPH? a. Cryoprecipitate b. Factor VIII and von Willebrand factor (vWf) c. Desmopressin d. Hemabate

C Desmopressin is the primary treatment of choice for vWD and can be administered orally, nasally, and intravenously. This medication promotes the release of factor VIII and vWf from storage. Cryoprecipitate may be used; however, because of the risk of possible donor viruses, other modalities are considered safer. Treatment with plasma products such as factor VIII and vWf is an acceptable option for this client. Because of the repeated exposure to donor blood products and possible viruses, this modality is not the initial treatment of choice. Although the administration of the prostaglandin, Hemabate, is known to promote contraction of the uterus during PPH, it is not effective for the client who has a bleeding disorder.

A woman who has recently given birth complains of pain and tenderness in her leg. On physical examination, the nurse notices warmth and redness over an enlarged, hardened area. Which condition should the nurse suspect, and how will it be confirmed? a. Disseminated intravascular coagulation (DIC); asking for laboratory tests b. von Willebrand disease (vWD); noting whether bleeding times have been extended c. Thrombophlebitis; using real-time and color Doppler ultrasound d. Idiopathic or immune thrombocytopenic purpura (ITP); drawing blood for laboratory analysis

C Pain and tenderness in the extremities, which show warmth, redness, and hardness, is likely thrombophlebitis. A Doppler ultrasound examination is a common noninvasive way to confirm the diagnosis. A diagnosis of DIC is made according to clinical findings and laboratory markers. With DIC, a physical examination will reveal symptoms that may include unusual bleeding, petechiae around a blood pressure cuff on the womans arm, and/or excessive bleeding from the site of a slight trauma such as a venipuncture site. Symptoms of vWD, a type of hemophilia, include recurrent bleeding episodes, prolonged bleeding time, and factor VIII deficiency. A risk for PPH exists with vWD but does not exhibit a warm or reddened area in an extremity. ITP is an autoimmune disorder in which the life span of antiplatelet antibodies is decreased. Increased bleeding time is a diagnostic finding, and the risk of postpartum uterine bleeding is increased.

The most effective and least expensive treatment of puerperal infection is prevention. What is the most important strategy for the nurse to adopt? a. Large doses of vitamin C during pregnancy b. Prophylactic antibiotics c. Strict aseptic technique, including hand washing, by all health care personnel d. Limited protein and fat intake

C Strict adherence by all health care personnel to aseptic techniques during childbirth and the postpartum period is extremely important and the least expensive measure to prevent infection. Good nutrition to control anemia is a preventive measure. Increased iron intake assists in preventing anemia. Antibiotics may be administered to manage infections; they are not a cost-effective measure to prevent postpartum infection. Limiting protein and fat intake does not help prevent anemia or prevent infection.

Changes in blood volume after childbirth depend on several factors such as blood loss during childbirth and the amount of extravascular water (physiologic edema) mobilized and excreted. What amount of blood loss does the postpartum nurse anticipate? (Select all that apply.) a. 100 ml b. 250 ml or less c. 300 to 500 ml d. 500 to 1000 ml e. 1500 ml or greater

C, D The average blood loss for a vaginal birth of a single fetus ranges from 300 to 500 ml (10% of blood volume). The typical blood loss for women who gave birth by cesarean is 500 to 1000 ml (15% to 30% of blood volume). During the first few days after childbirth, the plasma volume further decreases as a result of diuresis. Pregnancy-induced hypervolemia (i.e., an increase in blood volume of at least 35%) allows most women to tolerate considerable blood loss during childbirth.

After giving birth to a healthy infant boy, a primiparous woman, 16, is admitted to the postpartum unit. An appropriate nursing diagnosis for her at this time is "risk for impaired parenting related to deficient knowledge of newborn care." In planning for the woman's discharge, what should the nurse be certain to include in the plan of care? A. Tell the woman how to feed and bathe her infant B. Give the woman written information on bathing her infant C. Advise the woman that all mothers instinctively know how to care for their infants D. Provide time for the woman to bathe her infant after she views an infant bath demonstration

D A. Incorrect: Although verbalizing how to care for the infant is a form of client education, it is not the most developmentally appropriate teaching for a teenage mother. B. Incorrect: Although providing written information is useful, it is not the most developmentally appropriate teaching for a teenage mother. C. Incorrect: This statement is inappropriate; it is belittling and false. D. Correct: Having the mother demonstrate infant care is a valuable method of assessing the client's understanding of her newly acquired knowledge, especially in this age group, because she may inadvertently neglect her child. p. 625

After birth, a crying infant may be soothed by being held in a position in which the newborn can hear the mother's heartbeat. This phenomenon is known as: A. Entrainment B. Reciprocity C. Synchrony D. Biorhythmicity

D A. Incorrect: Entrainment is the movement of newborns in time to the structure of adult speech. B. Incorrect: Reciprocity is body movement or behavior that gives cues to the person's desires. These take several weeks to develop with a new baby. C. Incorrect: Synchrony is the fit between the infant's behavioral cues and the parent's responses. D. Correct: The newborn is in rhythm with the mother. The infant develops a personal biorhythm with the parents' help over time. p. 617

Nursing activities that promote parent-infant attachment are many and varied. One activity that should not be overlooked is the management of the environment. While providing routine mother-baby care, the nurse should ensure that: A. The baby is able to return to the nursery at night so that the new mother can sleep B. Routine times for care are established to reassure the parents C. The father should be encouraged to go home at night to prepare for mother-baby discharge D. An environment that fosters as much privacy as possible should be created

D A. Incorrect: Once the baby has demonstrated adjustment to extrauterine life (either in the mother's room or the transitional nursery), all care should be provided in one location. This important principle of family-centered maternity care fosters attachment by giving parents the opportunity to learn about their infant 24 hours a day. One nurse should provide care to both mother and baby in this couplet care or rooming-in model. It is not necessary for the baby to return to the nursery at night. In fact, the mother will sleep better with the infant close by. B. Incorrect: Care should be individualized to meet the parents' needs, not the routines of the staff. Teaching goals should be developed in collaboration with the parents. C. Incorrect: The father or other significant other should be permitted to sleep in the room with the mother. The maternity unit should develop policies that allow for the presence of significant others as much as the new mother desires. D. Correct: Care providers need to knock before gaining entry. Nursing care activities should be grouped. pp. 614-616

In follow-up appointments or visits with parents and their new baby, it is useful if the nurse can identify infant behaviors that can either facilitate or inhibit attachment. What is an inhibiting behavior? A. The infant cries only when hungry or wet. B. The infant's activity is somewhat predictable. C. The infant clings to the parents. D. The infant seeks attention from any adult in the room.

D A. Incorrect: These are facilitating behaviors. Facilitating and inhibiting behaviors are behaviors that build or discourage bonding (attitudes); they do not reflect any value judgments on what might be healthy or unhealthy. B. Incorrect: These are facilitating behaviors. Facilitating and inhibiting behaviors are behaviors that build or discourage bonding (attitudes); they do not reflect any value judgments on what might be healthy or unhealthy. C. Incorrect: These are facilitating behaviors. Facilitating and inhibiting behaviors are behaviors that build or discourage bonding (attitudes); they do not reflect any value judgments on what might be healthy or unhealthy. D. Correct: Parents want to be the focus of the infant's existence, just as the infant is the focus of their existence. Facilitating and inhibiting behaviors are behaviors that build or discourage bonding (attitudes); they do not reflect any value judgments on what might be healthy or unhealthy. p. 613

A 30-year-old multiparous woman has a boy who is 2 1/2 years old and now an infant girl. She tells the nurse, "I don't know how I'll ever manage both children when I get home." Which suggestion would best help this woman alleviate sibling rivalry? A. Tell the older child that he is a big boy now and should love his new sister B. Let the older child stay with his grandparents for the first 6 weeks to allow him to adjust to the newborn C. Ask friends and relatives not to bring gifts to the older sibling because you do not want to spoil him D. Realize that the regression in habits and behaviors in the older child is a typical reaction and that he needs extra love and attention at this time

D A. Incorrect: This strategy is a negative approach to facilitating sibling acceptance of the new infant. B. Incorrect: Reactions of siblings may result from temporary separation from the mother. Removing the older child from the home when the new infant arrives may enhance negative behaviors from the older child caused by separation from the mother. C. Incorrect: Providing small gifts from the infant to the older child is a strategy for facilitating sibling acceptance of the new infant. D. Correct: The older child may regress in habits or behaviors (e.g., toileting and sleep habits) as a method of seeking attention. Parents need to distribute their attention in an equitable manner. p. 629

If nonsurgical treatment for late PPH is ineffective, which surgical procedure would be appropriate to correct the cause of this condition? a. Hysterectomy b. Laparoscopy c. Laparotomy d. Dilation and curettage (D&C)

D D&C allows the examination of the uterine contents and the removal of any retained placental fragments or blood clots. Hysterectomy is the removal of the uterus and is not the appropriate treatment for late PPH. A laparoscopy is the insertion of an endoscope through the abdominal wall to examine the peritoneal cavity, but it, too, is not the appropriate treatment for this condition. A laparotomy is the surgical incision into the peritoneal cavity to explore it but is also not the appropriate treatment for late PPH.

Nurses need to understand the basic definitions and incidence data regarding PPH. Which statement regarding this condition is most accurate? a. PPH is easy to recognize early; after all, the woman is bleeding. b. Traditionally, it takes more than 1000 ml of blood after vaginal birth and 2500 ml after cesarean birth to define the condition as PPH. c. If anything, nurses and physicians tend to overestimate the amount of blood loss. d. Traditionally, PPH has been classified as early PPH or late PPH with respect to birth.

D Early PPH is also known as primary, or acute, PPH; late PPH is known as secondary PPH. Unfortunately, PPH can occur with little warning and is often recognized only after the mother has profound symptoms. Traditionally, a 500-ml blood loss after a vaginal birth and a 1000-ml blood loss after a cesarean birth constitute PPH. Medical personnel tend to underestimate blood loss by as much as 50% in their subjective observations.

The nurse suspects that her postpartum client is experiencing hemorrhagic shock. Which observation indicates or would confirm this diagnosis? a. Absence of cyanosis in the buccal mucosa b. Cool, dry skin c. Calm mental status d. Urinary output of at least 30 ml/hr

D Hemorrhage may result in hemorrhagic shock. Shock is an emergency situation during which the perfusion of body organs may become severely compromised, and death may occur. The presence of adequate urinary output indicates adequate tissue perfusion. The assessment of the buccal mucosa for cyanosis can be subjective. The presence of cool, pale, clammy skin is associated with hemorrhagic shock. Hemorrhagic shock is associated with lethargy, not restlessness.

Which classification of placental separation is not recognized as an abnormal adherence pattern? a. Placenta accreta b. Placenta increta c. Placenta percreta d. Placenta abruptio

D Placenta abruptio is premature separation of the placenta as opposed to partial or complete adherence. This classification occurs between the 20th week of gestation and delivery in the area of the decidua basalis. Symptoms include localized pain and bleeding. Placenta accreta is a recognized degree of attachment. With placenta accreta, the trophoblast slightly penetrates into the myometrium. Placenta increta is a recognized degree of attachment that results in deep penetration of the myometrium. Placenta percreta is the most severe degree of placental penetration that results in deep penetration of the myometrium. Bleeding with complete placental attachment occurs only when separation of the placenta is attempted after delivery. Treatment includes blood component therapy and, in extreme cases, hysterectomy may be necessary.

What is one of the initial signs and symptoms of puerperal infection in the postpartum client? a. Fatigue continuing for longer than 1 week b. Pain with voiding c. Profuse vaginal lochia with ambulation d. Temperature of 38 C (100.4 F) or higher on 2 successive days

D Postpartum or puerperal infection is any clinical infection of the genital canal that occurs within 28 days after miscarriage, induced abortion, or childbirth. The definition used in the United States continues to be the presence of a fever of 38 C (100.4 F) or higher on 2 successive days of the first 10 postpartum days, starting 24 hours after birth. Fatigue is a late finding associated with infection. Pain with voiding may indicate a urinary tract infection (UTI), but it is not typically one of the earlier symptoms of infection. Profuse lochia may be associated with endometritis, but it is not the first symptom associated with infection.

During which phase of maternal adjustment will the mother relinquish the baby of her fantasies and accept the real baby? a.Letting go c.Taking in b.Taking hold d.Taking on

NS: A Accepting the real infant and relinquishing the fantasy infant occurs during the letting-go phase of maternal adjustment. During the taking-hold phase the mother assumes responsibility for her own care and shifts her attention to the infant. In the taking-in phase the mother is primarily focused on her own needs. There is no taking-on phase of maternal adjustment.

Which description of postpartum restoration or healing times is accurate? a. The cervix shortens, becomes firm, and returns to form within a month postpartum. b. Vaginal rugae reappear by 3 weeks postpartum. c. Most episiotomies heal within a week. d. Hemorrhoids usually decrease in size within 2 weeks of childbirth.

b. Vaginal rugae reappear by 3 weeks postpartum. Vaginal rugae reappear by 3 weeks postpartum; however, they are never as prominent as in nulliparous women. The cervix regains its form within days; the cervical os may take longer. Most episiotomies take 2 to 3 weeks to heal. Hemorrhoids can take 6 weeks to decrease in size.

Which statement, related to the reconditioning of the urinary system after childbirth, should the nurse understand? a. Kidney function returns to normal a few days after birth. b. Diastasis recti abdominis is a common condition that alters the voiding reflex. c. Fluid loss through perspiration and increased urinary output accounts for a weight loss of more than 2 kg during the puerperium. d. With adequate emptying of the bladder, bladder tone is usually restored 2 to 3 weeks after childbirth.

c. Fluid loss through perspiration and increased urinary output accounts for a weight loss of more than 2 kg during the puerperium. Excess fluid loss through other means besides perspiration and increased urinary output occurs as well. Kidney function usually returns to normal in approximately 1 month. Diastasis recti abdominis is the separation of muscles in the abdominal wall and has no effect on the voiding reflex. Bladder tone is usually restored 5 to 7 days after childbirth.

Which documentation on a woman's chart on postpartum day 14 indicates a normal involution process? a. Moderate bright red lochial flow b. Breasts firm and tender c. Fundus below the symphysis and nonpalpable d. Episiotomy slightly red and puffy

c. Fundus below the symphysis and nonpalpable The fundus descends 1 cm per day; consequently, it is no longer palpable by postpartum day 14. The lochia should be changed by this day to serosa. Breasts are not part of the involution process. The episiotomy should not be red or puffy at this stage.

What should the nurse's next action be if the client's white blood cell (WBC) count is 25,000/mm3on her second postpartum day? a. Immediately inform the health care provider. b. Have the laboratory draw blood for reanalysis. c. Recognize that this count is an acceptable range at this point postpartum. d. Immediately begin antibiotic therapy.

c. Recognize that this count is an acceptable range at this point postpartum. During the first 10 to 12 days after childbirth, WBC values up to 30,000/mm3are common. Because a WBC count of 25,000/mm3on her second postpartum day is normal, alerting the health care provider is not warranted nor is reassessment or antibiotics needed; the WBC count is not elevated and so no reanalysis is required.

A client is concerned that her breasts are engorged and uncomfortable. What is the nurse's explanation for this physiologic change? a. Overproduction of colostrum b. Accumulation of milk in the lactiferous ducts and glands c. Hyperplasia of mammary tissue d. Congestion of veins and lymphatic vessels

d. Congestion of veins and lymphatic vessels Breast engorgement is caused by the temporary congestion of veins and lymphatic vessels. An overproduction of colostrum, an accumulation of milk in the lactiferous ducts and glands, and hyperplasia of mammary tissue do not cause breast engorgement.

The nurse is providing instruction to the newly delivered client regarding postbirth uterine and vaginal discharge, called lochia. Which statement is the most appropriate? a. Lochia is similar to a light menstrual period for the first 6 to 12 hours. b. It is usually greater after cesarean births. c. Lochia will usually decrease with ambulation and breastfeeding. d. It should smell like normal menstrual flow unless an infection is present.

d. It should smell like normal menstrual flow unless an infection is present. An offensive odor usually indicates an infection. Lochia flow should approximate a heavy menstrual period for the first 2 hours and then steadily decrease. Less lochia is usually seen after cesarean births and usually increases with ambulation and breastfeeding.

Which term best describes the interval between the birth of the newborn and the return of the reproductive organs to their normal nonpregnant state? a. Involutionary period because of changes to the uterus b. Lochia period because of the nature of the vaginal discharge c. Mini-tri period because it lasts only 3 to 6 weeks d. Puerperium, or fourth trimester of pregnancy

d. Puerperium, or fourth trimester of pregnancy the final period of pregnancy and lasts approximately 3 to 6 weeks. Involution marks the end of the puerperium. Lochia refers to the various vaginal discharges during the puerperium. Mini-tri period is not a commonly used term.


Kaugnay na mga set ng pag-aaral

Combo with "Accounting Chapter 15 test bank" and 3 others

View Set

MKT 2500 Exam 3 (Ch 6, 12, 13, 16, 17)

View Set

Nutrition: Lipids and Cardiovascular Disease

View Set

AMH2020 Ch. 26, CH 26- The Triumph of Conservativism, APUSH CH. 26

View Set

NetSuite: ERP Consultant Exam Review

View Set

NSG 310 - Foundations - Davis Edge for Lecture 11 (Ch 32, 34)

View Set

4 Fundamentals of Electricity: Unit 4 - Magnetism

View Set

Chapter 29: The Child with Musculoskeletal or Articular Dysfunction ANS

View Set